M

Chiudi

Home » Pillole teoriche integrali di linea

Pillole teoriche integrali di linea

    \[\,\]

    \[\,\]

    \[\,\]

Autori e revisori dell’articolo

    \[\,\]

    \[\,\]

Il presente lavoro è stato concepito come supporto pratico per affrontare gli esercizi relativi all’integrale di linea di seconda specie e all’analisi delle forme differenziali, con particolare attenzione ai loro domini. Nel corso dell’elaborato, si presuppone che il lettore abbia già acquisito una solida base teorica su questi argomenti, possibilmente studiando da un buon testo di riferimento1. Pertanto, la lettura è sconsigliata a chi non possiede ancora una conoscenza teorica consolidata, poiché, essendo strutturato come guida pratica, molti concetti verranno dati per acquisiti. Verranno citati solamente i teoremi necessari alla risoluzione degli esercizi.

L’esposizione seguirà un ordine che favorisce un approccio pratico, basato sulla logica del “problem solving”, lo stesso che dovrebbe adottare uno studente durante lo svolgimento di un esercizio. Queste pillole teoriche sono particolarmente utili per un corso di studi in Ingegneria o, in generale, per chi desidera approfondire la pratica di questi problemi.

    \[\,\]

    \[\,\]


  1. Ad esempio, si consigliano M. Bertsch, R. Dal Passo, L. Giacomelli, Analisi matematica o E. Giusti, Analisi matematica 2.

    \[\,\]

    \[\,\]

Definizioni utili

    \[\,\]

    \[\,\]

Leggi...

Forme differenziali

Si ricorda la definizione di forma differenziale.

    \[\,\]

Definizione 1.  Sia \Omega un aperto di \mathbb{R}^n e siano F_1(x_1,\dots,x_n),\dots, F_n(x_1,\dots,x_n) delle funzioni in \Omega. Si definisce forma differenziale la seguente espressione

    \[\omega=F_1(x_1,\dots,x_n)dx_1+\dots+F_n(x_1,\dots,x_n)dx_n=\sum_{j=1}^nF_j(x_1,\dots,x_n)dx_j.\]

Le funzioni F_1,\dots, F_n si dicono componenti di \omega. Una forma differenziale si dice continua su \Omega se le sue componenti sono funzioni continue su \Omega.

    \[\,\]

Nel caso in cui \Omega\subset\mathbb{R}^2 o \Omega\subset\mathbb{R}^3, adotteremo la seguenti notazione

    \begin{align*} \omega&=F_1(x,y)dx+F_2(x,y)dy\\ \omega&=F_1(x,y,z)dx+F_2(x,y,z)dy+F_3(x,y,z)dz. \end{align*}

Diamo di seguito due definizioni fondamentali che riguardano le forme differenziale che verranno usate nel seguito di questo testo

    \[\,\]

Definizione 2.  Sia \Omega\subseteq\mathbb{R}^3 un aperto connesso. Una forma differenziale \omega=F_1(x,y,z)\,dx+F_2(x,y,z)\,dy+F_3(x,y,z)\,dz continua in \Omega, associata al campo vettoriale F:\Omega \subseteq \mathbb{R}^3 \to \mathbb{R}^3 tale che F=\left(F_1(x,y,z),F_2(x,y,z),F_3(x,y,z)\right), si dice esatta in \Omega se esiste una funzione U: \Omega \to \mathbb{R}, tale che U \in \mathcal{C}^1(\Omega) e valga F =\overline{\nabla} U, cioè

    \[\begin{cases} F_1(x,y,z) = \dfrac{\partial U}{\partial x}(x,y,z)\\ \\ F_2(x,y,z) = \dfrac{\partial U}{\partial y}(x,y,z)\\ \\ F_3(x,y,z) = \dfrac{\partial U}{\partial z}(x,y,z). \end{cases}\]

In particolare U si definisce funzione potenziale di \omega o primitiva.

    \[\,\]

Definizione 3.  Sia una forma differenziale \omega=F_1(x,y,z)\,dx+F_2(x,y,z)\,dy+F_3(x,y,z)\,dz. Si definisce chiusa se è soddisfatta la seguente condizione (detta condizione delle derivate incrociate)

    \[\begin{cases} \dfrac{\partial F_3}{\partial y}(x,y,z)=\dfrac{ F_2}{\partial z}(x,y,z)\\\\ \dfrac{\partial F_1}{\partial z}(x,y,z)=\dfrac{\partial F_3}{\partial x}(x,y,z)\\\\ \dfrac{\partial F_2}{\partial x}(x,y,z)=\dfrac{\partial F_1}{\partial y}(x,y,z). \end{cases}\]

    \[\,\]

Con le definizioni precedenti risulta immediato verificare che una forma \omega è esatta allora essa risulta anche chiusa, in quanto si può applicare il teorema di Schwarz sulle derivate seconde miste. Nel seguito andremo invece ad analizzare quali sono le condizioni affinché una forma chiusa risulti esatta. Diamo infine la definitione di integrale di linea.

    \[\,\]

Definizione 4.  Sia r:[a,b]\rightarrow \mathbb{R}^n una curva di classe C^1 e per ogni (x_1,\dots,x_n)\in r([a,b]), sia \omega=F_1(x_1,\dots,x_n)dx_1+\dots+F_n(x_1,\dots,x_n)dx_n una forma differenziale continua. Si definisce integrale curvilineo di seconda specie

    \begin{align*} \int_{ r([a,b])}\omega&=\int_{ r([a,b])}\left(F_1dx_1+\dots+F_ndx_n\right)\\ &=\int_{a}^{b}\left(F_1\left( r(t)\right)r_1^\prime(t)+\dots+ F_n\left( r(t)\right)r_n^\prime(t)\right)\,dt. \end{align*}

    \[\,\]

    \[\,\]

    \[\,\]

    \[\,\]

Risultati fondamentali

    \[\,\]

    \[\,\]

Leggi...

Quando si ha a che fare con un integrale di linea di seconda specie di una forma differenziale o di un campo vettoriale, a seconda del linguaggio scelto, la prima cosa che al lettore viene spontanea fare è applicare la definizione: in realtà questa non è sempre la strada migliore da intraprendere. La prima cosa che è necessario fare è verificare se la forma differenziale sia chiusa oppure no.

    \[\,\]

    \[\,\]

Forma differenziale chiusa in un sottoinsieme di \mathbb{R}^3

    \[\,\]

Teorema 1.  Sia \Omega\subseteq \mathbb{R}^3 un insieme aperto e connesso. Una forma differenziale \omega=F_1\,dx+F_2\,dy+F_3\,dz di classe C^1\left(\Omega\right), associata al campo vettoriale F=\left(F_1,F_2,F_3\right), è chiusa in \Omega se vale \overline{\nabla} \wedgeF=\overline{0} (Rotore del campo vettoriale uguale a zero).

    \[\,\]

Si ricorda al lettore che il rotore di un campo vettoriale è

    \[\nabla \wedge F & = \begin{vmatrix}\hat{x} &\hat{y} &\hat{z} \\\\{\dfrac {\partial }{\partial x}}&{\dfrac {\partial }{\partial y}}&{\dfrac {\partial }{\partial z}}\\\\F_{1}&F_{2}&F_{3}\end{vmatrix}=\]

    \[= \hat{x} \left({\frac {\partial F_{3}}{\partial y}}-{\frac {\partial F_{2}}{\partial z}}\right)&+\hat{y} \left({\frac {\partial F_{1}}{\partial z}}-{\frac {\partial F_{3}}{\partial x}}\right)+\hat{z} \left({\frac {\partial F_{2}}{\partial x}}-{\frac {\partial F_{1}}{\partial y}}\right),\]

dove il determinante è inteso in senso formale e \hat x,\hat y,\hat z definiscono i versori nella direzione x,y e z, rispettivamente.

    \[\,\]

    \[\,\]

Forma differenziale chiusa in un sottoinsieme di \mathbb{R}^2

    \[\,\]

Nel caso di un sottoinsieme di \mathbb{R}^2 vale il seguente risultato che è una riformulazione del teorema 1.

    \[\,\]

Teorema 2.  Sia \Omega\subseteq \mathbb{R}^2 un insieme aperto e connesso. Una forma differenziale \omega=F_1\,dx+F_2\,dy di classe C^1\left(\Omega\right), associata al campo vettoriale \vec{F}=\left(F_1,F_2\right), è chiusa in \Omega se vale che le derivate miste sono uguali, ovvero se \dfrac{\partial F_1}{\partial y}=\dfrac{\partial F_2}{\partial x}.

    \[\,\]

Quindi, dal punto di vista teorico risulta chiaro come si determini quando una forma differenziale sia chiusa. Chiariamo il concetto con degli esempi.

    \[\,\]

Esempio 1.  Sia \omega=\dfrac{y}{\sqrt{1-x^2}}\,dx+\left(2y+\arcsin x\right)\,dy: verificare se è chiusa nel suo dominio.

    \[\,\]

Soluzione. Il dominio di \omega è \Omega=\{(x,y)\in\mathbb{R}^2:-1<x<1\}. Calcoliamo le derivate miste della forma differenziale

    \begin{equation*} \dfrac{\partial F_2}{\partial x}=\dfrac{1}{\sqrt{1-x^2}} \end{equation*}

e

    \begin{equation*} \dfrac{\partial F_1}{\partial y}=\dfrac{1}{\sqrt{1-x^2}} \end{equation*}

e osserviamo che

    \begin{equation*} \dfrac{\partial F_2}{\partial x}=\dfrac{\partial F_1}{\partial y} \end{equation*}

pertanto \omega è chiusa nel suo dominio.

    \[\,\]

Esempio 2.  Sia \omega= \left(y+z\right) \, dx + \left(x+z\right) \, dy+\left(x+y\right)\,dz=F_1\, dx +F_2 \,dy+F_3\,dz e verificare se è chiusa nel suo dominio.

    \[\,\]

Soluzione. Osserviamo che il suo dominio è \mathbb{R}^3. Calcoliamo il \nabla\wedge\vec{F} dove F=(F_1,F_2,F_3) è il campo vettoriale associato alla forma differenziale. Dunque

    \[\nabla\wedgeF&=\text{det}\begin{pmatrix} \hat{x} & \hat{y} & \hat{z}\]

    \[\dfrac{\partial }{\partial x} & \dfrac{\partial }{\partial y} & \dfrac{\partial }{\partial z}\]

    \[F_1& F_2& F_3 \end{pmatrix}= \\\\ =\hat{x}\left(\dfrac{\partial F_3}{\partial y}-\dfrac{\partial F_2}{\partial z}\right)&-\hat{y}\left(\dfrac{\partial F_3}{\partial x}-\dfrac{\partial F_1}{\partial z}\right)+\hat{z}\left(\dfrac{\partial F_2}{\partial x}-\dfrac{\partial F_1}{\partial y}\right)=\\\\ =\left(1-1\right)\,\hat{x}&-\hat{y}\left(1-1\right)+\hat{z}\left(1-1\right)=\vec{0}\]

dunque \omega è chiusa nel sul dominio.

    \[\,\]

    \[\,\]

Forma differenziale esatta

    \[\,\]

Data una forma differenziale, una volta stabilito se essa sia chiusa sul suo dominio, il passo successivo è quello di verificare la sua esattezza. A tale scopo ricordiamo il seguente risultato:

    \[\,\]

Teorema 3.  Sia \Omega\subseteq \mathbb{R}^n un insieme semplicemente connesso e \omega una forma differenziale chiusa in \Omega. Allora \omega è esatta in \Omega.

    \[\,\]

A questo punto forniamo dei metodi che permettono di applicare in maniera pratica il risultato appena esposto. Per fare ciò abbiamo ancora necessità di alcune definizioni di carattere topologico

    \[\,\]

    \[\,\]

Cenni di topologia di \mathbb{R}^n

    \[\,\]

    \[\,\]

Leggi...

    \[\,\]

Definizione 5.  Dati \overline{x}\in \mathbb{R}^n e \delta numero reale positivo non nullo, si dice intorno sferico di \overline{x} di raggio \delta l’insieme così definito

    \begin{equation*} I_\delta\left(\overline{x}\right):=\{\overline{y}\in\mathbb{R}^n:d(\overline{x},\overline{y})<\delta\}. \end{equation*}

    \[\,\]

Un altro modo di definire un intorno sferico di \overline{x} è pensarlo come una palla di centro \overline{x} e raggio \delta. Si vuole far osservare al lettore che in \mathbb{R}^2 l’intorno sferico non è nient’altro che un cerchio di centro \overline{x}=\left(x_0,y_0\right) e raggio \delta privato del bordo come in Figura 1.

    \[\,\]

    \[\,\]

    \[\,\]

Rendered by QuickLaTeX.com

    \[\,\]

    \[\,\]

    \[\,\]

Definizione 6.  Dati \overline{x}\in \mathbb{R}^n e \delta numero reale positivo non nullo, si definisce sfera di centro \overline{x} e raggio \delta

    \[S_\delta\left(\overline{x}\right)=\{\overline{y}\in\mathbb{R}^n:d\left(\overline{x},\overline{y}\right)=\delta\}.\]

    \[\,\]

In questo caso la rappresentazione grafica è quella data in Figura 2.

    \[\,\]

    \[\,\]

    \[\,\]

Rendered by QuickLaTeX.com

    \[\,\]

    \[\,\]

    \[\,\]

Definizione 7.  Sia \Omega \subseteq \mathbb{R}^n. Preso un punto di \overline{x}\in \mathbb{R}^n esso si dice:

    \[\,\]

  • interno ad \Omega, se esiste un intorno sferico di \overline{x} contenuto in \Omega;
  • esterno ad \Omega, se esiste un intorno sferico di \overline{x} contenuto nel complementare di \Omega;
  • di frontiera per \Omega, se ogni intorno sferico di \overline{x} contiene almeno un punto di \Omega e un punto del complementare di \Omega;

    \[\,\]

In Figura 4 rappresentiamo quanto detto

    \[\,\]

    \[\,\]

    \[\,\]

    \[\,\]

    \[\,\]

    \[\,\]

Definizione 8.  Un insieme A\subseteq\mathbb{R}^n si definisce

    \[\,\]

  • aperto se ogni punto è interno;
  • chiuso se il suo complementare1 è aperto.

        \[\,\]

    \[\,\]

Definizione 9.  Un insieme \Omega\subseteq\mathbb{R}^n si dice connesso se non è possibile trovare due insiemi entrambi aperti (o entrambi chiusi) A e B tali che \Omega=A\cup B e A\cap B=\emptyset.

    \[\,\]

Definizione 10.  Un insieme aperto \Omega\subseteq\mathbb{R}^n si definisce connesso per archi se, presi due punti di tale insieme, esiste un arco di curva continuo congiungente questi due punti interamente contenuto in \Omega.

    \[\,\]

Si può dimostrare che in \mathbb{R}^n le due definizioni 7 e 8 risultano equivalenti per insiemi aperti. Concludiamo questa sezione con quella che è la definizione topologica fondamentale per lo studio delle forme in \mathbb{R}^n. Ricordiamo che una curva \gamma:[a,b]\subseteq \mathbb{R}\rightarrow \mathbb{R}^2 si dice semplice se comunque presi due punti t_1,t_2\in (a,b), con t_1\neq t_2, risulta \gamma(t_1)\neq \gamma(t_2). Nel caso in cui \gamma(a)=\gamma(b), la curva semplice si dice chiusa.

    \[\,\]

Definizione 11.  Sia p\in\mathbb{R}^n un punto. Un arco (o laccio) centrato in p è una funzione continua f:[0,1]\to\mathbb{R}^n tale che f(0)=f(1)=p. Dati due lacci f,g centrati in p\in\mathbb{R}^n, un’omotopia da f a g è una funzione continua F:[0,1]\times[0,1]\to\mathbb{R}^n tale che F(t,0)=f(t) e F(t,1)=g(t), per ogni t\in[0,1]. Il laccio si dice contraibile se esiste un’omotopia che lo trasforma nel laccio costante g(t)\equiv p, per ogni t. Un sottoinsieme \Omega\subset\mathbb{R}^n connesso per archi si dice semplicemente connesso se per ogni p\in\Omega, ogni laccio centrato in p è contraibile.

    \[\,\]

    \[\,\]


  1. Se A\subseteq\mathbb{R}^n il suo complementare è l’insieme differenza \mathbb{R}^n\setminus A

    \[\,\]

    \[\,\]

Dominio semplicemente connesso: teoria ed applicazioni

    \[\,\]

    \[\,\]

Leggi...

Dominio semplicemente connesso nel piano

    \[\,\]

    \[\,\]

Nella maggior parte dei casi negli esercizi vengono trattati insiemi con dei “buchi”: si può ragionare in modo empirico e pensare che ogni tale sottoinsieme di \mathbb{R}^2 non sia un insieme semplicemente connesso. Infatti considerando una curva semplice e chiusa che circondi tale “buco” la sua contrazione porterà ad uscire dall’insieme. Forniamo un elenco di insiemi semplicemente connessi nel piano:

    \[\,\]

  • cerchio pieno, ellisse piena, poligono convesso pieno, semipiano e tutte le figure piane che si possono ottenere da queste deformandole con continuità; il piano stesso (\mathbb{R}^2) o il piano privato di una semiretta.

        \[\,\]

    Non sono semplicemente connessi

        \[\,\]

  • il piano o un cerchio privato di un punto (vale per una generica figura geometrica), poligoni concavi, una corona circolare, una circonferenza, ogni insieme che presenta un “buco”.

        \[\,\]

    Riportiamo alcuni esempi di esercizi in cui si mostra che una forma differenziale è esatta nel suo dominio applicando il Teorema 1.

        \[\,\]

    Esempio 3.  Verificare che

        \[\omega(x,y)= \dfrac{1+y}{1+x} \, dx + \ln(1+x) \, dy=F_1\, dx +F_2 \,dy\]

    è esatta nel suo insieme di definizione.

        \[\,\]

    Soluzione.

        \[\,\]

    Verifichiamo se la forma differenziale è chiusa. Calcolando le derivate miste

        \begin{equation*} \dfrac{\partial F_2}{\partial x} = \dfrac{1}{x+1} \quad \mbox{e} \quad \dfrac{\partial F_1}{\partial y} = \dfrac{1}{x+1} \end{equation*}

    osserviamo che sono uguali, quindi possiamo affermare che \omega è chiusa. Il dominio di \omega è

        \[\text{Dom} (\omega) = \{ (x,y) \in \mathbb{R}^2 \vert x>-1 \}\]

    che è un insieme semplicemente connesso (semipiano), allora \omega è esatta.

        \[\,\]

    Esempio 4.  Verificare che

        \[\omega(x,y)= e^y\sin\left(x+y\right) \, dx +e^y\left(\sin\left(x+y\right)-\cos\left(x+y\right)\right)\, dy=F_1\, dx +F_2 \,dy\]

    è esatta nel suo insieme di definizione.

        \[\,\]

    Soluzione.

        \[\,\]

    Verifichiamo se la forma differenziale è chiusa. Calcolando le derivate miste

        \begin{equation*} \dfrac{\partial F_2}{\partial x}=e^y\left(\cos\left(x+y\right)+\sin\left(x+y\right)\right) \end{equation*}

    e

        \begin{equation*} \dfrac{\partial F_1}{\partial y}=e^y\sin\left(x+y\right)+e^y\cos\left(x+y\right)=e^y\left(\cos\left(x+y\right)+\sin\left(x+y\right)\right) \end{equation*}

    osserviamo che sono uguali, quindi possiamo affermare che \omega è chiusa. Il dominio di \omega è \mathbb{R}^2 che è un insieme semplicemente connesso, allora \omega è esatta.

        \[\,\]

    Esempio 5.  Verificare che

        \[\omega(x,y)= y^2 \, dx + \left(\dfrac{1}{y}+2xy\right)\, dy=F_1\, dx +F_2 \,dy\]

    è esatta nel suo insieme di definizione applicando il teorema di Poincaré.

        \[\,\]

    Soluzione.

        \[\,\]

    Verifichiamo se la forma differenziale è chiusa. Calcolando le derivate miste

        \begin{equation*} \dfrac{\partial F_2}{\partial x} =2y \quad \mbox{e} \quad \dfrac{\partial F_1}{\partial y} = 2y \end{equation*}

    osserviamo che sono uguali, quindi possiamo affermare che \omega è chiusa. Il dominio di \omega è

        \[\text{Dom} (\omega) = \{ (x,y) \in \mathbb{R}^2 \vert\, y\neq0 \}\]

    che non è un insieme connesso e pertanto neanche semplicemente connesso (unione di due semipiani), quindi non possiamo applicare il teorema di Poincaré (Guardare Esempio 8 per maggiori dettagli). L’esempio 5 serve a sottolineare l’importanza del teorema di Poincaré e cosa accade quando non è possibile applicarlo: se la forma differenziale è chiusa, ma il suo dominio non è un insieme semplicemente connesso non possiamo dire nulla sull’esattezza di \omega nel suo dominio. Questo perché la condizione di semplice connessione è una condizione sufficiente ai fini del teorema.

        \[\,\]

    Esempio 6. Verificare se

        \[\omega(x,y)=\dfrac{x}{x+y} \; dx + \dfrac{y}{x+y} \; dy=F_1\,dx+F_2\,dy\]

    è esatta nel suo insieme di definizione.

        \[\,\]

    Soluzione.

        \[\,\]

    Il dominio di \omega è \left\{(x,y)\in \mathbb{R}^2:\, x \neq -y\right\}. Verifichiamo se \omega è chiusa procedendo come segue

        \[\left(\dfrac{\partial F_2}{\partial x} - \dfrac{\partial F_1}{\partial y} \right) =\]

        \[-y(x+y)^{-2} + x(x+y)^{-2}\neq 0.\]

    Dal risultato appena ottenuto, deduciamo che \omega non è chiusa e allora non è neanche esatta.

        \[\,\]

    Dall’esempio 6 è bene notare che se una forma differenziale non è chiusa, allora non è esatta; il fatto che una forma differenziale sia chiusa è una condizione necessaria per far sì che sia esatta.

        \[\,\]

        \[\,\]

    Dominio semplicemente connesso nello spazio

        \[\,\]

        \[\,\]

    Riportiamo ora degli esempi di insiemi semplicemente connessi nello spazio:

        \[\,\]

  • Sfera, ellissoide, poliedri convessi e tutti i solidi che si possono ottenere da questi ultimi deformandoli con continuità (sia i solidi, sia le loro frontiere), la corona sferica, il semispazio, lo spazio privo di un numero finito di punti.

        \[\,\]

    Non sono semplicemente connessi

        \[\,\]

  • l toro, la sfera privata di un diametro, lo spazio privato di una retta, poliedri concavi.

        \[\,\]

    Riportiamo un esempio

        \[\,\]

    Esempio 7.  Verificare che

        \[\omega(x,y,z)= \left(y+z\right) \, dx + \left(x+z\right) \, dy+\left(x+y\right)\,dz=F_1\, dx +F_2 \,dy+F_3\,dz\]

    è esatta nel suo insieme di definizione.

        \[\,\]

    Soluzione. Abbiamo già verificato nell’esempio 2 che la forma risulta chiusa. Inoltre osserviamo che il suo dominio è \mathbb{R}^3 che è un insieme semplicemente connesso, pertanto per il teorema di Poincaré \omega è esatta.

        \[\,\]

    \[\,\]

    \[\,\]

Forma differenziale localmente esatta

    \[\,\]

    \[\,\]

Leggi...

    \[\,\]

Abbiamo imparato a capire quando una forma differenziale è chiusa oppure no; abbiamo inoltre dato delle condizioni per verificare quando la chiusura implica l’esattezza. Tuttavia nei casi pratici non è detto che questo accada sempre poiché una buona percentuale di esercizi presenta forme differenziali chiuse che non hanno come dominio un insieme semplicemente connesso: in questo caso il Teorema di Poincaré non è efficace e non possiamo concludere nulla. Esiste, in ogni caso, un importante corollario del Teorema di Poincaré che viene in aiuto in queste situazioni.

    \[\,\]

Corollario 1.  Sia \omega una forma differenziale chiusa in un aperto \Omega qualunque (anche non connesso). Ogni punto di \Omega ha un intorno sferico I_\delta contenuto in \Omega e I_\delta è semplicemente connesso. Allora in I_\delta la forma differenziale è localmente esatta, ovvero per ogni x\in\Omega esiste un intorno U di x tale che I_\delta è esatta su U.

    \[\,\]

Riportiamo un esempio del Corollario 1.

    \[\,\]

Esempio 8.  Verificare che

    \[\omega(x,y)= y^2 \, dx + \left(\dfrac{1}{y}+2xy\right)\, dy=F_1\, dx +F_2 \,dy\]

è esatta nel suo insieme di definizione .

    \[\,\]

Soluzione. Abbiamo già visto nell’esempio 5 che la forma è chiusa. Il dominio di \omega è

    \[\text{Dom} (\omega) = \{ (x,y) \in \mathbb{R}^2 \vert\, y\neq0 \}\]

che non è un insieme connesso e pertanto neanche semplicemente connesso, quindi non possiamo dire nulla sull’esattezza di \omega nel suo dominio secondo il Teorema di Poincaré. Osserviamo però che il dominio di \omega può essere riscritto come segue

    \[\text{Dom} (\omega)=\underbrace{\{(x,y)\in\mathbb{R}^2:y>0\}}_{D_1}\cup\underbrace{\{(x,y)\in\mathbb{R}^2:y<0\}}_{D_2}\]

dove sia D_1 che D_2 sono insiemi semplicemente connessi. Quindi sicuramente \omega è localmente esatta in [slider]ciascuno [/slider] dei due insiemi.

    \[\,\]

    \[\,\]

Condizioni di equivalenza sulle forme esatte

    \[\,\]

    \[\,\]

Enunciamo il seguente teorema che esprime condizioni di equivalenza sulle forme esatte. Ricordiamo che una curva chiusa \gamma:I\subset\mathbb{R}\rightarrow \mathbb{R}^3 si dice regolare a tratti quando esiste una partizione dell’intervallo I, \{I_1,\ldots,I_k\} con I=\bigcup_{i=1}^{k}I_i e I_i\cap I_j=\emptyset, i\neq j, tale che \gamma risulta regolare su ogni I_i.

    \[\,\]

Teorema 4.  Siano \Omega\subseteq \mathbb{R}^3 un aperto connesso e \omega una forma differenziale di classe C\left(\Omega\right). Allora le seguenti affermazioni sono equivalenti:

    \[\,\]

  1. Per ogni coppia di curve regolari a tratti \gamma_1,\gamma_2 contenute in \Omega ed aventi stesso punto iniziale e stesso punto finale,

        \[\int_{\gamma_1} \omega = \int_{\gamma_2} \omega\]

  2. Per ogni curva chiusa semplice \gamma, regolare a tratti e contenuta in \Omega,

        \[\oint_{\gamma} \omega = 0\]

  3. \omega esatta.

    \[\,\]

Sostanzialmente, il Teorema 4 stabilisce modi equivalenti di esprimere il concetto di esattezza di una forma differenziale: una forma differenziale è esatta se e solo se l’integrale di linea di seconda specie lungo qualsiasi curva chiusa (detto anche circuitazione nel linguaggio dei campi vettoriali) è nullo, oppure se e solo se l’integrale di linea tra due punti qualsiasi non dipende dal percorso seguito ma solo dai punti di partenza ed arrivo. Riportiamo di seguito alcuni esempi che racchiudono gli ultimi concetti spiegati.

    \[\,\]

Esempio 9.  Calcolare il seguente integrale di seconda specie

(1)   \begin{equation*} \int_{\gamma}\left(2xy^3+3\right)\,dx+\left(3x^2y^2\right)\,dy=\int_{\gamma}F_1\,dx+F_2\,dy=\int_{\gamma}\omega \end{equation*}

dove il sostegno di \gamma è rappresentato da una circonferenza centrata nell’origine, di raggio 1, percorsa in senso antiorario.

    \[\,\]

Soluzione. Calcoliamo le derivate miste della forma differenziale

    \begin{equation*} \dfrac{\partial F_2}{\partial x}=6xy^2 \qquad \mbox{e} \qquad \dfrac{\partial F_1}{\partial y}=6xy^2. \end{equation*}

Osserviamo che

    \begin{equation*} \dfrac{\partial F_2}{\partial x}= \dfrac{\partial F_1}{\partial y} \end{equation*}

pertanto \omega è chiusa. Inoltre il suo dominio è un insieme semplicemente connesso quindi \omega è esatta nel suo dominio; facilmente concludiamo, grazie al punto 2 del teorema 4 che

    \[\int_{\gamma}\omega=0.\]

    \[\,\]

Definizione 10.  Calcolare il seguente integrale di seconda specie

(2)   \begin{equation*} I=\int_{\gamma}-\dfrac{y}{x^2+y^2}\,dx+\dfrac{x}{x^2+y^2}\, dy=\int_{\gamma}F_1\,dx+F_2\,dy=\int_{\gamma}\omega \end{equation*}

dove il sostegno di \gamma è dato dalla circonferenza di equazione \left(x-2\right)^2+y^2=1 percorsa in senso antiorario.

    \[\,\]

Soluzione. Rappresentiamo il sostegno di \gamma

    \[\,\]

    \[\,\]

    \[\,\]

Rendered by QuickLaTeX.com

    \[\,\]

    \[\,\]

    \[\,\]

Calcoliamo le derivate miste

    \begin{equation*} \dfrac{\partial F_2}{\partial x}=\dfrac{x^2+y^2-x(2x)}{\left(x^2+y^2\right)^2 }=\dfrac{y^2-x^2}{\left(x^2+y^2\right)^2} \end{equation*}

e

    \begin{equation*} \dfrac{\partial F_1}{\partial y}=-\dfrac{x^2+y^2-y\left(2y\right)}{\left(x^2+y^2\right)^2}=\dfrac{y^2-x^2}{\left(x^2+y^2\right)^2} \end{equation*}

e osserviamo che

    \begin{equation*} \dfrac{\partial F_2}{\partial x}=\dfrac{\partial F_1}{\partial y}, \end{equation*}

pertanto \omega è chiusa, ma come si può osservare il dominio è \mathbb{R}^2\setminus\{(0,0)\} che non è un insieme semplicemente connesso, quindi non possiamo dire nulla sull’esattezza di \omega. Osserviamo però che la circonferenza si trova in una zona del dominio che è un insieme semplicemente connesso, quindi è localmente esatta ed essendo un integrale di linea su un percorso chiuso possiamo concludere che I=0.

    \[\,\]

Osserviamo che anche se non sappiamo se la forma differenziale è esatta o meno sul suo dominio \mathbb{R}^2-\{(0,0)\}, l’esempio 10 pu\`o essere svolto lo stesso in modo immediato dopo aver verificato che \omega è chiusa. Infatti, il sostegno della curva lungo la quale dobbiamo calcolare l’integrale di linea è interamente contenuto in una porzione semplicemente connessa del dominio (per esempio, l’insieme \Omega' = \{(x,y) \in \mathbb{R}^2 : x > 1/2\}). La forma differenziale è quindi localmente esatta, e, dato che il sostegno di \gamma è una curva chiusa, l’integrale è nullo.

    \[\,\]

    \[\,\]

Calcolo di un potenziale per una forma differenziale esatta

    \[\,\]

    \[\,\]

Leggi...

A questo punto della nostra trattazione ricordiamo un teorema che risulterà fondamentale nella risoluzione degli esercizi, come mostreremo negli esempi che seguiranno.

    \[\,\]

Teorema 5.  Siano \Omega \subseteq \mathbb{R}^n un aperto connesso ed \omega una forma differenziale continua ed esatta in \Omega. Sia poi \gamma una curva di classe C^1 a tratti, contenuta in \Omega, con parametrizzazione r\left(t\right) del sostegno di \gamma con t\in \left[a,b\right] e con punti iniziale e finale rispettivamente r\left(a\right) e r\left(b\right). Allora

    \[\int_\gamma\omega=U( r(b))-U( r(a))\]

dove U è una funzione potenziale di \omega.

    \[\,\]

Alla luce di questo ultimo risultato, l’ultimo passo prima di procedere con gli esempi fondamentali, è quello di comprendere come ottenere un potenziale data la forma differenziale da integrare.

    \[\,\]

Esempio 11.  Calcolare il seguente integrale di seconda specie

(3)   \begin{equation*} \int_{\gamma}\dfrac{y}{\sqrt{1-x^2}}\,dx+\left(2y+\arcsin x\right)\,dy=\int_{\gamma}F_1\,dx+F_2\,dy=\int_{\gamma}\omega \end{equation*}

dove il sostegno di \gamma è rappresentata dalla poligonale aperta di vertici \left(\frac{1}{2},1\right),\left(-\frac{1}{2},0\right) e \left(0,3\right) percorsa nell’ordine illustrato.

    \[\,\]

Soluzione. Rappresentiamo il sostegno di \gamma

    \[\,\]

    \[\,\]

    \[\,\]

Rendered by QuickLaTeX.com

    \[\,\]

    \[\,\]

    \[\,\]

Calcoliamo le derivate miste della forma differenziale

    \begin{equation*} \dfrac{\partial F_2}{\partial x}=\dfrac{1}{\sqrt{1-x^2}} \end{equation*}

e

    \begin{equation*} \dfrac{\partial F_1}{\partial y}=\dfrac{1}{\sqrt{1-x^2}} \end{equation*}

e osserviamo che

    \begin{equation*} \dfrac{\partial F_2}{\partial x}=\dfrac{\partial F_1}{\partial y} \end{equation*}

pertanto \omega è chiusa nel suo dominio, esso è l’insieme

    \[\Omega=\{(x,y)\in\mathbb{R}^2:-1<x<1\},\]

che è semplicemente connesso: ne risulta che \omega è esatta. Per poter calcolare un potenziale U è necessario che siano soddisfatte le seguente condizioni:

(4)   \begin{equation*} \begin{cases} \dfrac{\partial U}{\partial x}=\dfrac{y}{\sqrt{1-x^2}}\\\\ \dfrac{\partial U}{\partial y}=2y+\arcsin x \end{cases}. \end{equation*}

Dalla prima delle due equazioni precedenti otteniamo, integrando rispetto ad x,

    \begin{equation*} U(x,y)=y\arcsin x+c(y), \end{equation*}

che sostituita nella seconda conduce a

    \begin{equation*} \arcsin x +\dfrac{d c}{dy}(y)=2y+\arcsin x \quad \Leftrightarrow \quad c(y)=y^2+k. \end{equation*}

Concludiamo quindi che

    \begin{equation*} U(x,y)=y^2+y\arcsin x+k. \end{equation*}

Grazie a quanto fatto fino ad ora, possiamo calcolare 3 come segue

    \begin{equation*} \int_\gamma\omega=U\left(0,3\right)-U\left(\frac{1}{2},1\right)=9-\left(1+\dfrac{\pi}{6}\right)=8-\dfrac{\pi}{6} \end{equation*}

e pertanto

    \[\,\]

 

    \[\int_{\gamma}\omega=8-\dfrac{\pi}{6}.\]

    \[\,\]

Esempio 12.  Verificare che

    \[\omega(x,y,z)= \left(y+z\right) \, dx + \left(x+z\right) \, dy+\left(x+y\right)\,dz=F_1\, dx +F_2 \,dy+F_3\,dz\]

è esatta nel suo insieme di definizione e in caso affermativo trovare un potenziale.

    \[\,\]

Soluzione. Abbiamo già verificato che nell’esempio 2 che è chiusa e nel 7 che è esatta. Calcoliamo un potenziale: esso deve soddisfare le condizioni

(5)   \begin{equation*} \begin{cases} \dfrac{\partial U}{\partial x} = y+z\\\\ \dfrac{\partial U}{\partial y} = x+z\\\\ \dfrac{\partial U}{\partial z}=x+y \end{cases}. \end{equation*}

Dalla prima equazione otteniamo

    \begin{equation*} U(x,y,z)=\left(y+z\right)x+c_1(y,z) \end{equation*}

che sostituita nella seconda equazione conduce a

    \begin{equation*} \dfrac{\partial U}{\partial y}=x+\dfrac{\partial c_1}{\partial y}\left(y,z\right)=x+z \quad \Leftrightarrow \quad c_1(y,z)=yz+c_2(z). \end{equation*}

Infine

    \begin{equation*} U(x,y,z)=yx+zx+yz+c_2\left(z\right) \end{equation*}

che sostituita nella terza equazione porta

    \begin{equation*} \dfrac{\partial U}{\partial z}=x+y+\dfrac{dc_2}{dz}(z)=x+y \quad \Leftrightarrow \quad c_2\left(z\right)=k \end{equation*}

dove k è una costante. Si conclude che il potenziale è

    \[\,\]

 

    \[U(x,y,z)=xy+xz+yz+\text{k}.\]

    \[\,\]

    \[\,\]

Confronto fra i linguaggi per le forme differenzali ed i campi vettoriali

    \[\,\]

    \[\,\]

Leggi...

    \[\,\]

Sottolineamo per il lettore che gli integrali di seconda specie possono essere formulati in due modi differenti: con il linguaggio delle forme differenziali o quello dei campi vettoriali. La scelta dipende dal contesto e dal tipo di applicazione che se ne fa: non è scopo di questa guida pretendere di spiegare le differenze e i perché di tali scelte, per cui ci limiteremo a riportare una tabella presa dal libro “Analisi matematica vol.2, Bramanti-Pagani-Salsa”, per illustrare le diverse tipologie di tali linguaggi:

    \[\,\]

    \[\,\]

    \[\,\]

Rendered by QuickLaTeX.com

    \[\,\]

    \[\,\]

    \[\,\]

Al lettore proponiamo una riflessione. Se volessimo dimostrare che una forma differenziale chiusa non è esatta, come si potrebbe fare? solitamente si cerca una curva chiusa tale per cui l’integrale di linea di seconda specie su essa risulti diverso da zero: questo porta a concludere che la forma differenziale non è esatta o che il campo non è conservativo; in alternativa si può determinare il potenziale e verificare che esso sia di classe C^1 nel dominio della forma differenziale, in questo modo si dimostra che è esatta.

    \[\,\]

Esempio 13.  Sia

    \[F=\left(-\dfrac{y}{x^2+y^2}, \dfrac{x}{x^2+y^2}\right)=\left(F_1,F_2\right).\]

Verificare che F è conservativo nel suo dominio ed in caso trovare un potenziale.

    \[\,\]

Soluzione. Verifichiamo se F è irrotazionale. Calcolando le derivate miste, osserviamo che la loro differenza è pari a zero:

    \begin{align*} \dfrac{\partial F_2 }{\partial x}-\dfrac{\partial F_1}{\partial y}&=\dfrac{x^2+y^2-x\left(2x\right)}{\left(x^2+y^2\right)^2}-\left(-\dfrac{x^2+y^2-y\left(2y\right)}{\left(x^2+y^2\right)^2}\right)=\\ &=\dfrac{y^2-x^2}{\left(x^2+y^2\right)^2}-\dfrac{y^2-x^2}{\left(x^2+y^2\right)^2}=0, \end{align*}

pertanto F è irrotazionale nel suo dominio. Inoltre il dominio di F è \mathbb{R}^2\setminus\{(0,0)\} che non è un insieme semplicemente connesso, quindi non possiamo usare il Lemma di Poincarè. Cerchiamo un potenziale U che deve soddisfare le condizioni

(6)   \begin{equation*} \begin{cases} \dfrac{\partial U}{\partial x}=F_1=-\dfrac{y}{x^2+y^2}\\\\ \dfrac{\partial U}{\partial y}=F_2=\dfrac{x}{x^2+y^2}. \end{cases} \end{equation*}

Dalla prima equazione otteniamo

    \begin{equation*} U(x,y)=\int -\dfrac{y}{x^2+y^2}\,dx=-\int \dfrac{\frac{1}{y}}{1+\frac{x^2}{y^2}}\,dx=-\arctan\left(\dfrac{x}{y}\right)+c(y) \end{equation*}

e sostituendo quest’ultimo risultato nella seconda equazione si ha

    \begin{equation*} \frac{x}{x^2+y^2}+\dfrac{dc}{dy}(y)=\dfrac{x}{x^2+y^2} \quad \Leftrightarrow \quad c(y)=\text{costante}. \end{equation*}

Si conclude che il potenziale è

    \[\,\]

    \[U(x,y)=-\arctan\left(\dfrac{x}{y}\right)+\text{costante}.\]

    \[\,\]

Osserviamo che U non è di classe C^1 nel dominio di F, pertanto F non è conservativo. Si poteva altresì procedere cercando una curva chiusa \gamma regolare a tratti contenuta all’interno del dominio del campo vettoriale tale che

    \begin{equation*} \int_{\gamma}F\cdot d s\neq0. \end{equation*}

Proviamo a dimostrare che il campo vettoriale non è conservativo scegliendo una superficie chiusa \gamma avente come sostegno una circonferenza di raggio 1 centrata nell’origine e percorsa in senso antiorario

    \[\,\]

Rendered by QuickLaTeX.com

    \[\,\]

La sua parametrizzazione è

    \[\,\]

    \begin{equation*} r(\theta)=\left(\cos \theta ,\sin \theta\right)\quad \text{con}\,\, \theta \in \left[0,2\pi\right] \end{equation*}

mentre

    \begin{equation*} r^\prime(\theta)=\left(-\sin \theta,\cos \theta\right) \end{equation*}

è la sua derivata. Restringiamo F lungo r\left(\theta\right)

    \begin{align*} F\left( r(\theta)\right)&= \left(-\dfrac{\sin\theta}{\left(\cos^2\theta+\sin^2\theta\right)},\dfrac{\cos\theta}{\left(\cos^2\theta+\sin^2\theta\right)}\right)=\\ &=\left(-\sin\theta,\cos\theta\right) \end{align*}

da cui3

    \begin{align*} \int_{\gamma}F\left( r\left(t\right)\right)\cdot r^\prime\left(t\right)\, dt &=\int_{0}^{2\pi} \left(-\sin \theta,\cos \theta\right) \cdot \left(-\sin\theta,\cos \theta\right)\, d \theta=\\ &=\int_{0}^{2\pi}\left(\cos^2\theta+\sin^2\theta\right)\, d \theta=2\pi\neq0. \end{align*}

Abbiamo dunque trovato una curva chiusa lungo la quale l’integrale di linea di seconda specie del campo vettoriale F è diverso da zero, quindi F non è conservativo.

    \[\,\]

    \[\,\]


  1. Dato un campo vettoriale

        \[F:A \subset \mathbb{R}^3 \rightarrow \mathbb{R}^3\]

    tale che F(x,y,z)=(F_1(x,y,z),F_2(x,y,z),F_3(x,y,z)) e una curva regolare o regolare a tratti \gamma definita in A con parametrizzazione

        \[r:[a,b]\rightarrow \mathbb{R}^3\]

    tale che r(t)=(x(t),y(t),z(t)), si definisce integrale di linea di seconda specie:

    (7)   \begin{equation*} \int_\gamma F\cdot d s=\int_{a}^{b}F\left( r(t) \right)\cdot r ^\prime (t)\, dt. \end{equation*}

    \[\,\]

    \[\,\]

Combinazioni lineari di forme differenziali esatte e non esatte

    \[\,\]

    \[\,\]

Leggi...

    \[\,\]

Abbiamo imparato come calcolare un integrale di linea di seconda specie. Come si ragiona? Innanzitutto cerchiamo di capire se la forma è chiusa e di conseguenza se è esatta nel suo dominio o localmente esatta; quindi cerchiamo di sfruttare il potenziale, valutandolo nel punto finale e iniziale della nostra curva, facendone la differenza e ottenendo così il valore dell’integrale. Nel caso in cui la forma differenziale non sia chiusa e di conseguenza non esatta siamo costretti ad applicare la definizione di integrale di linea di seconda specie. Spesso accade che le forme differenziali non siano esatte ma si possono scrivere come la combinazione lineare di forme differenziali esatte e non esatte e pertanto in questo modo si semplificano i calcoli. Riportiamo alcuni esempi di forme differenziali che non sono esatte ma si possono scrivere come la combinazione lineare di forme differenziali esatte con altre che non lo sono.

    \[\,\]

Esempio 14.  Calcolare il seguente integrale di seconda specie

(8)   \begin{equation*} I=\int_{\partial^+ A}y\,dx-x\, dy=\int_{\partial^+A}F_1\,dx+F_2\,dy \end{equation*}

essendo A il dominio tratteggiato in figura e \partial^+A è il bordo di A percorso una volta in senso antiorario.

    \[\,\]

    \[\,\]

    \[\,\]

Rendered by QuickLaTeX.com

    \[\,\]

    \[\,\]

    \[\,\]

Soluzione.

    \[\,\]

Riscriviamo 8 come segue

(9)   \begin{equation*} I=\int_{\partial^+A}\underbrace{y\,dx+x\,dy}_{\omega}-2\underbrace{\int_{\partial^+A}x\,dy}_{I_1}. \end{equation*}

Si osserva facilmente che \omega è esatta, pertanto \int_{\partial ^+A}\omega=0. Non ci resta che calcolare I_1. Vogliamo calcolare i punti di intersezione tra la funzione y=1/(1+x^2) e y=1/2, pertanto imponiamo la seguente equazione

    \begin{equation*} \dfrac{1}{1+x^2}=\dfrac{1}{2} \quad \Leftrightarrow \quad x^2+1=2 \quad \Leftrightarrow \quad x =\pm 1 \end{equation*}

da cui si evince che gli estremi del segmento rappresentato in figura sono (-1,1/2) e (1,1/2). Rappresentiamo il sostegno di \partial^+A dato da \gamma_1\cup \gamma_2:

    \[\,\]

    \[\,\]

    \[\,\]

Rendered by QuickLaTeX.com

    \[\,\]

    \[\,\]

    \[\,\]

dove \gamma_1 è il segmento che congiunge i punti (-1,1/2) e (1,1/2) e \gamma_2 è il grafico di y=1/(1+x^2) con x \in [-1,1]. Possiamo riscrivere I_1 come segue

(10)   \begin{equation*} I_1=\int_{\gamma_1}x\,dy+\int_{\gamma_2}x\,dy. \end{equation*}

Parametrizziamo il sostegno di \gamma_1 come

    \begin{equation*} r_1\left(t\right)=\left(t,\dfrac{1}{2}\right)\quad \text{con}\,\, t \in[-1,1] \end{equation*}

e deriviamo quanto ottenuto

    \begin{equation*} r_1^\prime(t)=\left(1,0\right) \end{equation*}

da cui deduciamo \int_{\gamma_1}x\,dy=0.\\\\ Parametrizziamo ora il sostegno di \gamma_2

    \begin{equation*} r_2\left(t\right)=\left(t,\dfrac{1}{1+t^2}\right)\quad \text{con}\,\, t \in\left[-1,1\right] \end{equation*}

ottenendo

    \begin{equation*} r^\prime_2\left(t\right)=\left(1,-\dfrac{2t}{\left(1+t^2\right)^2}\right). \end{equation*}

È importante osservare che abbiamo parametrizzato r_2 in senso orario per comodità: successivamente, nel calcolo dell’integrale di linea della forma differenziale x\,dy sul sostegno di \gamma_2 metteremo un meno davanti all’integrale. Procediamo dunque come segue

    \begin{align*} &\int_{\gamma_2}x\,dy=-\int_{-1}^{1}t\cdot \dfrac{-2t}{\left(1+t^2\right)^2}\,dt=\int_{-1}^{1} \dfrac{2t^2}{\left(1+t^2\right)^2}\,dt= \end{align*}

Sfruttando il fatto che data una funzione f(x) pari in un intervallo [-a,a], l’integrale su tale intervallo della funzione f può essere riscritto come \int_{-a}^{a}f(x)\,dx=2\int_{0}^{a}f(x)\,dx

    \begin{align*} &=4\int_{0}^{1}\dfrac{t^2+1-1}{\left(1+t^2\right)^2}\,dt=4\left(\int_{0}^{1}\dfrac{1+t^2}{\left(1+t^2\right)^2}\,dt-\int_{0}^{1}\dfrac{1}{\left(1+t^2\right)^2}\,dt\right)=\\ &=4\left(\int_{0}^{1}\dfrac{1}{1+t^2}\,dt-\int_{0}^{1}\dfrac{1}{\left(1+t^2\right)^2}dt\right)= \end{align*}

utilizzando l’integrale indefinito notevole \int \frac{f^\prime(x)}{1+f^2(x)}\,dx=\arctan\left(f(x)\right)+c (dove in questo caso f(x) = x) per calcolare il primo integrale e la sostituzione \tan t=z4 per il secondo,

    \begin{align*} &=4\left(\arctan t \bigg \vert^1_0-\int_{0}^{\frac{\pi}{4}}\dfrac{1}{\left(1+\tan^2 t\right)^2}\cdot \left(1+\tan^2 t\right)\,dt\right)=\\ &=4\left(\dfrac{\pi}{4}-\int_{0}^{\frac{\pi}{4}}\dfrac{1}{1+\tan^2 t }\, dt=\right)=\pi-4\int_{0}^{\frac{\pi}{4}}\cos^2 t \, dt= \end{align*}

utilizzando l’identità \cos^2 t = \frac{1+\cos\left(2t\right)}{2}

    \begin{align*} &=\pi-4\int_{0}^{\frac{\pi}{4}}\dfrac{1+\cos\left(2t\right)}{2}\,dt=\pi-2\int_{0}^{\frac{\pi}{4}}1\,dt-2\int_{0}^{\frac{\pi}{4}}\cos\left(2t\right)\,dt=\\ &=\pi-\dfrac{\pi}{2}-\sin\left(2t\right)\bigg \vert^{\frac{\pi}{4}}_0=\dfrac{\pi}{2}-1 \end{align*}

Concludiamo che

    \begin{equation*} I_1=\dfrac{\pi}{2}-1 \end{equation*}

per cui l’integrale diventa

    \begin{equation*} I=-2\left(\dfrac{\pi}{2}-1\right)=2-\pi. \end{equation*}

Concludiamo che

    \[\,\]

 

    \[I=2-\pi.\]

    \[\,\]

Esempio 15.  Calcolare

(11)   \begin{equation*} \int_{\gamma} \omega \end{equation*}

dove

    \[\omega = \left(6x - \dfrac{2xy}{(x^2+2)^2} \right) \; dx + \left(\dfrac{1+6x+3x^3}{x^2+2}\right) \; dy\]

e \gamma è l’arco di circonferenza nel piano (x,y) che va da (1,0) a (0,1) passante per (2,1).

    \[\,\]

Soluzione.

    \[\,\]

Riscriviamo l’integrale come

    \begin{align*} & \int_{\partial^+A} \left(6x - \dfrac{2xy}{(x^2+2)^2} \right) \; dx + \left(\dfrac{1+6x+3x^3}{x^2+2}\right) \; dy =\\ & \qquad = \int_{\partial^+A} 6x \; dx - \dfrac{2xy}{(x^2+2)^2} \; dx + \dfrac{1}{x^2+2} \; dy + 3x \; dy \end{align*}

e chiamiamo F_1 = 6x, \displaystyle F_2=-\frac{2xy}{(x^2+2)^2}, \displaystyle F_3 = \frac{1}{x^2+2} e F_4 = 3x . Osserviamo che

    \begin{equation*} \dfrac{\partial F_1}{\partial y} = 0, \qquad \dfrac{\partial F_2}{\partial y} = -\dfrac{2x}{(x^2+2)^2}, \qquad \dfrac{\partial F_3}{\partial x}= \dfrac{-2x}{(x^2+2)^2}\quad \mbox{e} \quad \dfrac{\partial F_4}{\partial x}=3. \end{equation*}

Pertanto definendo

    \begin{equation*} \omega_1 = (F_1 + F_2) \, dx + F_3 \, dy \end{equation*}

si può osservare che essa ha come insieme di definizione \Omega=\mathbb{R}^2 che è un insieme semplicemente connesso: essendo \omega_1 chiusa, essa risulta esatta in \Omega. Per calcolare l’integrale di linea abbiamo bisogno di capire com’è fatto il sostegno di \partial^+A, pertanto conoscendo i tre punti che appartengono alla circonferenza, imponiamo il seguente sistema5

    \begin{equation*} \begin{cases} 1+a+c=0\\1+b+c=0\\4+1+2a+b+c=0 \end{cases}\quad \Leftrightarrow \quad \begin{cases} a=-2\\b=-2\\c=1 \end{cases} \end{equation*}

ottenendo

    \begin{equation*} x^2+y^2-2x-2y+1=0. \end{equation*}

La circonferenza appena trovata ha centro C(1,1) e raggio r=1 come rappresentato nella seguente figura (indichiamo anche il verso di percorrenza della circonferenza)

    \[\,\]

    \[\,\]

    \[\,\]

Rendered by QuickLaTeX.com

    \[\,\]

    \[\,\]

    \[\,\]

Calcoliamo un potenziale U per \omega_1 : esso soddisfa le condizioni

(12)   \begin{equation*} \begin{cases} \dfrac{\partial U}{\partial x} = F_1+F_2 = 6x - \dfrac{2xy}{(x^2+2)^2}\\\\ \dfrac{\partial U}{\partial y} = F_3= \dfrac{1}{x^2+2} \end{cases} \end{equation*}

Dalla prima equazione otteniamo

    \begin{equation*} U(x,y) =\int\left(6 x -\dfrac{2xy}{(x^2+2)^2}\right)\,dx= 3x^2+\dfrac{y}{x^2+2}+c(y). \end{equation*}

Sostituiamo U(x,y) nella seconda equazione

    \begin{equation*} \dfrac{\partial U}{\partial y} = \dfrac{1}{x^2+2}+\dfrac{\partial c(y)}{\partial y}=\dfrac{1}{x^2+2} . \end{equation*}

da cui

    \begin{equation*} \dfrac{\partial c(y)}{\partial y} = 0 \quad \Leftrightarrow \quad c(y) = \mbox{cost}. \end{equation*}

Ne risulta

    \begin{equation*} U(x,y) = 3x^2+\dfrac{y}{x^2+2}+\mbox{cost}, \end{equation*}

e dunque possiamo calcolare 11 come

    \begin{equation*} \int_\gamma \omega_1 = U(0,1)-U(1,0) = \dfrac{1}{2}-3 = -\dfrac{5}{2}. \end{equation*}

Consideriamo ora

    \begin{equation*} \int_{\gamma} 3x \, dy= \int_{\gamma} \omega_2. \end{equation*}

dove la parametrizzazione di \gamma è

    \begin{equation*} r(\theta) = \left( 1+\sin \theta, 1- \cos \theta\right), \qquad \mbox{con } \theta \in \left[0,\dfrac{3}{2}\pi\right]. \end{equation*}

Allora6

    \begin{align*} \int_\gamma\omega_2&=\int_{0}^{\frac{3}{2}\pi}3 \left(\sin \theta +1\right)\sin \theta\, d \theta=3\int_{0}^{\frac{3}{2}\pi}\left(\sin^2\theta+\sin \theta\right)\, d \theta=\\ &=3\int_{0}^{\frac{3}{2}\pi}\left(\dfrac{1-\cos\left(2 \theta\right)}{2}+\sin \theta\right)\, d \theta=\\ &=\dfrac{3}{2}\int_{0}^{\frac{3}{2}\pi}d\theta-\dfrac{3}{2}\int_{0}^{\frac{3}{2}\pi}\cos \left(2\theta\right)\, d \theta+3\int_{0}^{\frac{3}{2}\pi}\sin \theta\, d \theta=\\ &=\dfrac{9}{4}\pi-\dfrac{3}{4}\sin\left(2\theta\right)\bigg\vert^{\frac{3}{2}\pi}_{0}-3\cos\theta\bigg\vert^{\frac{3}{2}\pi}_0=\dfrac{9}{4}\pi+3. \end{align*}

Pertanto 11 diventa

    \[\,\]

 

    \[\int_{\gamma} \omega = -\dfrac{5}{2}+\dfrac{9}{4}\pi+3 = \dfrac{9}{4}\pi +\dfrac{1}{2}.\]

    \[\,\]

    \[\,\]


  1. Nello svolgimento continua a comparire la variabile t e non z poiché è variabile muta.

    \[\,\]

  1. Si ricorda che l’equazione generale di una circonferenza nel piano è x^2+y^2+ax+by+c=0 con a,b,c \in \mathbb{R}.
  1. Dalla formula di duplicazione del coseno si ricava: \sin^2\theta=\frac{1-\cos\left(2 \theta\right)}{2} che tornerà utile nel calcolo dell’integrale.

    \[\,\]

    \[\,\]

Applicazione della definizione di integrale di linea di seconda specie di una forma differenziale

    \[\,\]

    \[\,\]

Leggi...

Continuiamo la nostra discussione con un esempio in cui applichiamo la definizione di integrale di linea senza verificare se precedentemente se la forma differenziale sia esatta oppure se il campo vettoriale sia conservativo.

    \[\,\]

Esempio 16.  Calcolare applicando la definizione di integrale di linea di seconda specie

(13)   \begin{equation*} \int_{\gamma} F \cdot d\bm{r} \end{equation*}

dove

    \[F> = (y^4+e^{xy}y)\hat{x} + (4y^3x+xe^{xy})\hat{y}\]

e il sostegno di \gamma è la spezzata di vertici (0,0), (1,0), (1,-1) e (2,-2) percorsa nell’ordine in cui sono stati forniti i punti.

    \[\,\]

Soluzione. Rappresentiamo graficamente il sostegno di \gamma

    \[\,\]

    \[\,\]

    \[\,\]

Rendered by QuickLaTeX.com

    \[\,\]

    \[\,\]

    \[\,\]

e osserviamo che \gamma può essere riscritta come

    \[\gamma=\gamma_1 \cup \gamma_2 \cup \gamma_3\]

dove il sostegno di \gamma_1 è il segmento che congiunge i punti (0,0) e (1,0), il sostegno di \gamma_2 è il segmento che congiunge i punti (1,0) e (1,-1) e il sostegno di \gamma_3 è il segmento che congiunge i punti (1,-1) e (2,-2). Sia r_1(t) la parametrizzazione di \gamma_1, r_2(t) la parametrizzazione di \gamma_2 e r_3(t) la parametrizzazione di \gamma_3. La rappresentazione analitica è quella che segue:

    \[& r_1(t) = (t,0), \qquad &&\mbox{con } \, t \in [0,1]\]

    \[& r_2(t) = (1,-t), \qquad &&\mbox{con } \, t \in [0,1]\]

    \[& r_3 (t)= (t,-t), \qquad &&\mbox{con } \, t \in [1,2].\]

Restringiamo \bf{F} lungo le tre parametrizzazioni r_1(t), r_2(t)\,\, \text{e}\,\, r_3(t) e successivamente eseguiamo il prodotto scalare con la derivata delle tre parametrizzazioni:

    \[&F(r_1(t)) \cdot {r_1}^\prime(t) = F( r_1(t)) \cdot (1,0) = 0\]

    \[& F( r_2(t)) \cdot { r_2}^\prime(t) = F( r_2(t)) \cdot (0,-1) = 4t^3-e^{-t}\]

    \[&F( r_3(t)) \cdot { r_3}^\prime(t) =F( r_3(t)) \cdot (1,-1) = 5t^4-2te^{-t^2}\]

    \[\end{aligned}\]

Possiamo ora calcolare 15 applicando la definizione di integrale di linea di seconda specie:

    \[& \int_0^1 F( r_1) \cdot { r_1}^\prime(t) \, dt + \int_0^1 F( r_2) \cdot { r_2}^\prime(t) \, dt + \int_1^2 F( r_3) \cdot { r_3}^\prime(t) \, dt =\]

    \[& = \int_0^1 \left( 4t^3-e^{-t}\right) \, dt+ \int_1^2 \left( 5t^4-2te^{-t^2}\right)\, dt =\]

    \[&=\left(t^4+e^{-t} \right)\bigg \vert^1_0+\left(t^5+e^{-t^2}\right)\bigg \vert^2_1 =31+e^{-4}.\]

Pertanto

    \[\,\]

 

    \[\int_{\gamma} F \cdot d \bm{\ell}=31+e^{-4}.\]

    \[\,\]

Cerchiamo di capire ora quando risulta conveniente applicare la definizione versus utilizzare i teoremi enunciati all’inizio di questo manuale. Riprendiamo l’esempio 16 e verifichiamo se \bf{F} sia irrotazionale. Poiché

    \[\dfrac{\partial F_2}{\partial x} - \dfrac{\partial F_1}{\partial y}=4y^3+e^{xy}+xye^{xy}-4y^3-e^{xy}-xye^{xy}=0\]

possiamo affermare che F è irrotazionale. Inoltre il dominio di F è \mathbb{R}^2 che è un insieme semplicemente connesso, dunque F è conservativo in \mathbb{R}^2. Calcoliamo un potenziale U ricorrendo alla definizione

    \[F = \nabla U = \left(\dfrac{\partial U}{\partial x} ,\dfrac{\partial U}{\partial y} \right)\]

da cui

(14)   \begin{equation*} \begin{cases} \dfrac{\partial U}{\partial x} = y^4 + y \, e^{xy}\\\\ \dfrac{\partial U}{\partial y} = 4xy^3 + x \, e^{xy}. \end{cases} \end{equation*}

Dalla prima equazione abbiamo

    \[U(x,y)=y^4x+e^{xy}+\mbox{c}(y),\]

che sostituita nella seconda conduce a

    \[\dfrac{\partial U}{\partial y}=4xy^3 + x \, e^{xy}+c'(y)=4xy^3 + x \, e^{xy}\]

da cui si ricava che

    \[\mbox{c}^\prime(y)=0 \quad \Leftrightarrow \quad \mbox{c}(y)=\mbox{cost}.\]

Allora il potenziale è

    \[\,\]

 

    \[U(x,y)=y^4x+e^{xy}+\mbox{cost}.\]

    \[\,\]

Ora basta valutare il potenziale nel punto finale e iniziale e facendo la differenza di tali valori otteniamo il risultato ottenuto in precedenza. Dove sta il vantaggio di procedere in questo modo? Per prima cosa non è stato necessario cercare una parametrizzazione in quanto sono sufficienti le coordinate del punto iniziale e finale; inoltre non bisogna passare per il calcolo dei prodotti scalari ed eventuali semplificazioni. Tuttavia ciò che risulta di maggiore economia è il fatto di evitare di calcolare un certo numero di integrali che (sebbene in questo caso presentassero solo funzioni potenza ed esponenziale) potrebbero risultare molto complessi e di non facile soluzione.

    \[\,\]

    \[\,\]

Forme differenziali esatte con parametro

    \[\,\]

    \[\,\]

Di seguito si propongono degli esempi dove al variare di un parametro reale \alpha si richiede se è possibile determinare tale parametro affinché la forma differenziale sia esatta nel suo dominio.

    \[\,\]

Esempio 17.  Sia

    \[\omega=\left(\frac{y}{1+xy}+\alpha y\right)\,dx+\left(\frac{x}{1+xy}+x\right)\,dy=F_1\,dx+F_2\,dy\]

con F_1 e F_2 continue in \Omega=\{(x,y)\in \mathbb{R}^2:\,x>0,\,y>0\} e \alpha numero reale. Determinare se esistono valori di \alpha affinché

  • \omega sia esatta in \Omega;
  • \int_{\gamma}\omega =0 dove il sostegno di \gamma è il segmento che congiunge i punti (2,2) e (3,3).
  •     \[\,\]

    Soluzione.

        \[\,\]

    Verifichiamo se è possibile trovare il valore di \alpha affinché \omega sia chiusa nel suo dominio \Omega. Calcoliamo le derivate miste

        \begin{equation*} \dfrac{\partial F_2}{\partial x}=\dfrac{1+xy-xy}{\left(1+xy\right)^2}+1=\dfrac{1}{\left(1+xy\right)^2}+1 \end{equation*}

    e

        \begin{equation*} \dfrac{\partial F_1}{\partial y}=\dfrac{1+xy-yx}{\left(1+xy\right)^2}+\alpha=\dfrac{1}{\left(1+xy\right)^2}+\alpha \end{equation*}

    da cui

        \begin{equation*} \dfrac{\partial F_2}{\partial x}=\dfrac{\partial F_1}{\partial y}\quad \Leftrightarrow \quad \alpha=1. \end{equation*}

    Concludiamo che \omega è chiusa se e solo se \alpha=1 e inoltre osserviamo che \Omega è un insieme semplicemente connesso, pertanto per il Teorema di Poincaré concludiamo che \omega è esatta in \Omega. Calcoliamo il valore di \alpha affinché risulti \int_\gamma \omega=0 applicando la definizione. Parametrizziamo il sostegno di \omega

        \begin{equation*} r\left(t\right)=\left(t,t\right)\quad \text{con}\,\, t \in \left[2,3\right] \end{equation*}

    e

        \begin{equation*} r^\prime\left(t\right)=\left(1,1\right) \end{equation*}

    la derivata di r\left(t\right). Pertanto

        \begin{align*} \int_{\gamma}\omega&=\int_{2}^{3}\left(\dfrac{t}{1+t^2}+\alpha t+\dfrac{t}{1+t^2}+t\right)\,dt=\\ &=\int_{2}^{3}\left(\dfrac{2t}{1+t^2}+\left(\alpha+1\right)t\right)\,dt=\ln\left(1+t^2\right)+\dfrac{\left(\alpha+1\right)}{2}t^2\bigg \vert^3_2=\\ &=\ln 10 +\dfrac{9}{2}\left(\alpha+1\right)-\ln 5- 2\left(\alpha +1\right)=\ln 2 +\dfrac{5}{2}\left(\alpha+1\right)=0 \quad \Leftrightarrow \quad \end{align*}

        \begin{equation*} \quad \Leftrightarrow \quad \alpha=-\dfrac{2}{5}\ln 2 -1. \end{equation*}

        \[\,\]

    Esempio 18.  Sia

    (15)   \begin{equation*} \omega:=\left(\alpha x z+\dfrac{yz}{x}\right)\,dx+\left(z\ln x-\dfrac{\alpha^2 y}{2}\ln z\right)\,dy+\left(x^\alpha+y\ln x-\dfrac{y^2}{z}\right)\,dz. \end{equation*}

    Calcolare per quali valori di \alpha è esatta e per tali valori di \alpha calcolare la primitiva di \omega.

        \[\,\]

    Soluzione.

        \[\,\]

    Siano F_1(x,y,z):=\alpha x z+\dfrac{yz}{x}, F_2(x,y,z):=z\ln x-\dfrac{\alpha^2 y}{2}\ln z e F_3(x,y,z):=x^\alpha+y\ln x-\dfrac{y^2}{z}. Verifichiamo se la forma differenziale è chiusa

        \begin{align*} & \dfrac{\partial F_3}{\partial y}=\dfrac{\partial F_2}{\partial z} \quad \Leftrightarrow \qquad \ln x-\dfrac{2y}{z}=\ln x-\dfrac{\alpha^2 y}{2z} \quad \Leftrightarrow \quad \alpha=2,\\ & \dfrac{\partial F_1}{\partial z}=\dfrac{\partial F_3}{\partial x}\quad \Leftrightarrow \quad \alpha x +\dfrac{y}{x}=\alpha x^{\alpha-1}+\dfrac{y}{x}\quad \Leftrightarrow \quad \alpha=2,\\ &\dfrac{\partial F_2}{\partial x} =\dfrac{\partial F_1}{\partial y} \quad \Leftrightarrow \quad \dfrac{z}{x}=\dfrac{z}{x} \quad \Leftrightarrow \quad \forall \alpha \in \mathbb{R} \end{align*}

    da cui si conclude che la forma differenziale è chiusa se e solo se \alpha=2. Ora notiamo che il dominio della nostra forma differenziale è \left\{(x,y,z) \in \mathbb{R}^3\, : x>0,\, z>0 \right\} che è un insieme semplicemente connesso: allora per il teorema di Poincaré possiamo concludere che è esatta nel suo dominio. Possiamo calcolare il potenziale ponendo \alpha=2

    (16)   \begin{equation*} \begin{cases} F_1 = \dfrac{\partial U}{\partial x}\\ \\ F_2 = \dfrac{\partial U}{\partial y}\\ \\ F_3 = \dfrac{\partial U}{\partial z}. \end{cases} \end{equation*}

    Dalla prima si ottiene

        \begin{align*} U(x,y,z)&=\int\left(2 x z +\dfrac{y z}{x}\right)\,dx=\dfrac{2 x^2 z}{2}+y z \ln \left \vert x \right \vert +c(y,z)\overset{x>0}{=}\\ &\overset{x>0}{=}x^2 z+y z \ln \left( x \right) +c(y,z) \end{align*}

    e sostituendo U nella seconda abbiamo

        \begin{align*} \dfrac{\partial U}{\partial y}&=z\ln x+\dfrac{\partial c}{\partial y}\left(y,z\right)=z \ln x -2y\ln z \quad \Leftrightarrow \quad \\ &\quad \Leftrightarrow \quad c(y,z)=-2\ln z\int y\,dy=- y^2 \ln z+g(z) \end{align*}

    da cui

        \begin{equation*} U(x,y,z)=x^2 z+yz\ln x-y^2\ln z+g(z). \end{equation*}

    Sostituiamo ora U nella terza equazione

        \begin{align*} \dfrac{\partial U}{\partial z}=x^2+y\ln x-\dfrac{y^2}{z}+\dfrac{d g}{d z}\left(z\right)=x^2+y\ln x -\dfrac{y^2}{z} \quad \Leftrightarrow \quad g(z)=\text{c}\in\mathbb{R}. \end{align*}

    Si conclude che il potenziale è

        \[\,\]

     

        \begin{equation*} U(x,y,z)=x^2z+yz\ln x-y^2\ln z+\text{c}. \end{equation*}

  •     \[\,\]

        \[\,\]

    Forme differenziali e coordinate polari

        \[\,\]

        \[\,\]

    Leggi...

        \[\,\]

    In questa sezione proponiamo alcuni esercizi dove è utile ricorrere al passaggio in coordinate polari per la parametrizzazione dei domini e delle curve.

        \[\,\]

    Esempio 24.  Calcolare il seguente integrale di seconda specie

    (17)   \begin{equation*} I=\int_{+\Gamma}\left(x^2+y^2-x\right)\,dy \end{equation*}

    dove il sostegno di \Gamma è la cardioide in figura avente come equazione polare

    (18)   \begin{equation*} \rho=1+\cos\theta \quad \text{con}\,\, \theta \in \left[0,2\pi\right]. \end{equation*}

        \[\,\]

    Soluzione.

        \[\,\]

    Rendered by QuickLaTeX.com

        \[\,\]

    Parametrizziamo il sostegno di \Gamma come segue

        \begin{align*} r\left(\theta\right)&=\left(x(\theta),y(\theta)\right)=\left(\rho \cos \theta,\rho \sin\theta\right)=\\ &=\left(\left(1+\cos\theta\right)\cos\theta,\left(1+\cos\theta\right)\sin\theta\right)=\\ &=\left(\cos\theta+\cos^2\theta,\sin\theta+\cos\theta\sin\theta\right)=\\ &=\left(\cos\theta+\cos^2\theta,\sin\theta+\dfrac{1}{2}\sin\left(2\theta\right)\right) \end{align*}

    e calcoliamo la derivata di r\left(\theta\right)

        \begin{align*} r^\prime\left(\theta\right)&=\left(-\sin\theta-2\cos\theta\sin\theta,\cos\theta+\cos\left(2\theta\right)\right)=\\ &=\left(-\sin\theta-\sin\left(2\theta\right),\cos\theta+\cos\left(2\theta\right)\right). \end{align*}

    Calcoliamo 17 applicando la definizione di integrale di linea

        \begin{align*} I&=\int_{0}^{2\pi}\left(\left(1+\cos\theta\right)^2-\cos\theta\left(1+\cos\theta\right)\right)\left(\cos\theta+\cos\left(2\theta\right)\right)\,d\theta=\\ &=\int_{0}^{2\pi}\left(1+\cos^2\theta+2\cos\theta-\cos\theta-\cos^2\theta\right)\left(\cos\theta+\cos\left(2\theta\right)\right)\,d\theta=\\ &=\int_{0}^{2\pi}\left(1+\cos\theta\right)\left(\cos\theta+1-2\sin^2\theta\right)\,d\theta=\\ &=\int_{0}^{2\pi}\left(\cos\theta+1-2\sin^2\theta+\cos^2\theta+\cos\theta-2\sin^2\theta\cos\theta\right)\,d\theta=\\ &=2\int_{0}^{2\pi}\cos\theta\,d\theta+\int_{0}^{2\pi}1\,d\theta-2\int_{0}^{2\pi}\sin^2\theta\,d\theta+\int_{0}^{2\pi}\cos^2\theta\,d\theta+\\ & \qquad -2\int_{0}^{2\pi}\sin^2\theta\cos\theta\,d\theta\overset{\clubsuit}{=}\\ &\overset{\clubsuit}{=}2\pi-2\int_{0}^{2\pi}\dfrac{1-\cos\left(2\theta\right)}{2}\,d\theta+\int_{0}^{2\pi}\dfrac{1+\cos\left(2\theta\right)}{2}\,d\theta -\dfrac{2}{3}\sin^3\theta\bigg\vert^{2\pi}_0=\\ &=2\pi-\int_{0}^{2\pi}1\,d\theta+\int_{0}^{2\pi}\cos\left(2\theta\right)\,d\theta+\dfrac{1}{2}\int_{0}^{2\pi}1\,d\theta+\\ &\qquad +\dfrac{1}{2}\int_{0}^{2\pi}\cos\left(2\theta\right)\,d\theta\overset{\diamond}{=}\\ &\overset{\diamond}{=}2\pi-2\pi+\pi=\pi \end{align*}

    dove in \clubsuit abbiamo usato il fatto che l’integrale del coseno sul periodo è nullo e le seguenti formule notevoli: \displaystyle\sin^2\theta=\frac{1+\cos\left(2\theta\right)}{2},\displaystyle\cos^2\theta=\frac{1-\cos\left(2\theta\right)}{2}; inoltre in \diamond abbiamo sfruttato il fatto che l’integrale del coseno su un multiplo del suo periodo è nullo. Dunque concludiamo che

        \[\,\]

     

        \[I=\pi.\]

        \[\,\]

    Esempio 25.  Calcolare applicando la definizione di integrale di linea di seconda specie

    (19)   \begin{equation*} \int_{\gamma} F \cdot d\bm{r} \end{equation*}

    dove

        \begin{equation*} F=\left(-y,x\right) \end{equation*}

    e il sostegno di \gamma è rappresentato dalla seguente equazione in forma polare

    (20)   \begin{equation*} \rho=R\,\theta\quad \text{con}\,\, \theta\in \left[0,4\pi\right] \end{equation*}

    dove R>0 è un parametro.

        \[\,\]

    Soluzione.

        \[\,\]

    Parametrizziamo il sostegno di una generica curva nel piano in coordinate polari

    (21)   \begin{equation*} r\left(\rho,\theta\right)=\left(\rho\cos \theta,\rho \sin\theta\right). \end{equation*}

    Sostituiamo 20 in 21 ottenendo così

    (22)   \begin{equation*} r\left(R\,\theta,\theta\right)=\bm{\psi}\left(\theta\right)=R\left(\theta\cos\theta,\theta\sin\theta\right) \quad \text{con}\,\, \theta\in\left[0,4\pi\right]. \end{equation*}

    Deriviamo 22 rispetto a \theta

        \begin{equation*} \bm{\psi}^\prime\left(\theta\right)=R\left(\cos\theta-\theta\sin\theta,\sin\theta+\theta\cos\theta\right) \end{equation*}

    e restringiamo F lungo \bm{\psi}\left(\theta\right)

        \begin{equation*} F\left(\bm{\psi}\left(\theta\right)\right)=R\left(-\theta\sin\theta,\theta\cos\theta\right). \end{equation*}

    Calcoliamo il prodotto scalare tra F>\left(\bm{\psi}\left(\theta\right)\right) e \bm{\psi}^\prime\left(\theta\right)

        \begin{align*} F\left(\bm{\psi}\left(\theta\right)\right)&\cdot \bm{\psi}^\prime\left(\theta\right)=\\ & =R^2\left(-\theta\sin\theta,\theta\cos\theta\right)\cdot\left(\cos\theta-\theta\sin\theta,\sin\theta+\theta\cos\theta\right) =\\ &=R^2\left(-\theta\cos\theta\sin\theta+\theta^2\sin^2\theta+\theta\cos\theta\sin\theta+\theta^2\cos^2\theta\right)=\\ &=R^2\left(\theta^2\left(\cos^2\theta+\sin^2\theta\right)\right)=R^2\theta^2. \end{align*}

    Abbiamo dunque

        \begin{equation*} \int_{\gamma}F\cdot d \bm{\ell}=R^2\int_{0}^{4\pi}\theta^2\,d \theta=R^2\cdot\dfrac{\theta^3}{3}\bigg\vert^{4\pi}_0=\dfrac{64}{3}\pi^3R^2. \end{equation*}

    Si conclude che

        \[\,\]

    div style=”padding: 10px; background-color: #afeeee;”>  

        \[\int_{\gamma}F\cdot d \bm{\ell}=\dfrac{64}{3}\pi^3R^2.\]

        \[\,\]

        \[\,\]

    Teorema di Gauss-Green

        \[\,\]

        \[\,\]

    Leggi...

        \[\,\]

    In questa sezione analizziamo il Teorema di Gauss-Green e le sue applicazioni.

        \[\,\]

    Teorema di Gauss-Green.  Sia A\subset \mathbb{R}^2 un aperto non vuoto e limitato tale che la frontiera di A sia una curva regolare a tratti e siano f_1,f_2 \in C^1(A). Allora data la forma differenziale

        \[\omega=f_1(x,y)dx+f_2(x,y)dy\]

    vale la seguente formula:

        \[\int_{+\partial A}\omega=\iint_A\left( \dfrac{\partial f_2}{\partial x}(x,y)-\dfrac{\partial f_1}{\partial y}(x,y)\right) \,dxdy\]

    dove +\partial A è la frontiera di A orientata positivamente (in senso antiorario).

        \[\,\]

        \[\,\]

        \[\,\]

    Rendered by QuickLaTeX.com

        \[\,\]

        \[\,\]

        \[\,\]

    Spesso negli esercizi è utile applicare il teorema di Gauss-Green, ad esempio quando abbiamo l’integrale di linea di una forma differenziale chiusa su un sostegno di una curva che circuita un “buco”.

        \[\,\]

    Esempio 19.  Calcolare

    (23)   \begin{equation*} \int_{\gamma} \omega \end{equation*}

    dove

    (24)   \begin{equation*} \omega = (3x + y) \, dx + xy \, dy \end{equation*}

    e il sostegno di \gamma è dato, nell’ordine, dal segmento di estremi (1,1) e (2,2), dalla semicirconferenza di estremi (2,2) e (0,0) passante per (0,2) e dal segmento di estremi (0,0) e (2,0).

        \[\,\]

    Soluzione.

        \[\,\]

    È immediato osservare che (1,1) è il punto medio del segmento che congiunge i punti (0,0) e (2,2) che è il diametro della semicirconferenza. Rappresentiamo graficamente il sostegno di \gamma

        \[\,\]

        \[\,\]

        \[\,\]

    Rendered by QuickLaTeX.com

        \[\,\]

        \[\,\]

        \[\,\]

    dove il sostegno di \gamma_1 è il segmento che congiunge i punti da (1,1) a (2,2), quello di \gamma_2 è la semicirconferenza e infine quello di \gamma_3 è il segmento che congiunge i punti da (0,0) a (2,0). Riscriviamo 24 come segue

        \begin{equation*} \omega=\omega_1+\omega_2 \end{equation*}

    dove

        \begin{equation*} \omega_1 = F_1 \, dx = 3x \quad \text{e}\quad \omega_2 = F_2 \, dx + F_3 \, dy = y \, dx + xy \, dy. \end{equation*}

    Il dominio di \omega è \mathbb{R}^2, che è un insieme semplicemente connesso: calcoliamo le derivate miste per vedere se \omega è chiusa nel suo dominio

        \begin{align*} & \dfrac{\partial F_1}{\partial y}=0, \qquad \dfrac{\partial F_2}{\partial y}=1 \quad \mbox{e} \quad \dfrac{\partial F_3}{\partial x}=y. \end{align*}

    Notiamo che le derivate miste sono diverse pertanto \omega non è chiusa. Osserviamo che \omega_1 è chiusa ed inoltre, dal momento che il suo dominio è un insieme semplicemente connesso, è esatta, quindi possiamo calcolare un potenziale U:

        \begin{equation*} \dfrac{dU}{dx}=3x \, dx \quad \Leftrightarrow \quad U = \dfrac{3}{2}x^2+\mbox{c} \end{equation*}

    con c costante. Calcoliamo l’integrale di linea di \omega_1 sul sostegno \gamma

        \begin{equation*} \int_\gamma \omega_1 = U(2,0)-U(1,1)=\dfrac{3}{2}\left(4-1\right) = \dfrac{9}{2}. \end{equation*}

    Non ci resta che calcolare l’integrale di linea di \omega_2 sul sostegno di \gamma=\gamma_1\cup\gamma_2\cup\gamma_3:

        \begin{equation*} \int_{\gamma}\omega_2=\int_{\gamma_1}\omega_2+\int_{\gamma_2}\omega_2+\int_{\gamma_3}\omega_2. \end{equation*}

    Parametrizziamo il sostegno di \gamma_1

        \begin{equation*} r_1(t) = (t,t) \quad \text{con}\,\, t \in [1,2] \end{equation*}

    da cui

        \begin{align*} \int_{\gamma_1} \omega_2 & = \int_1^2 (t,t^2) \cdot (1,1) \, dt = \\ & =\int_{1}^{2}\left(t+t^2\right)\,dt=\dfrac{t^2}{2}+\dfrac{t^3}{3}\bigg\vert^2_1=\\ &=\dfrac{4}{2}+\dfrac{8}{3}-\dfrac{1}{2}-\dfrac{1}{3}= \dfrac{23}{6}. \end{align*}

    Ora parametrizziamo il sostegno di \gamma_27

        \begin{equation*} r_2(\theta) = (\sqrt{2}\cos \theta+1,\sqrt{2}\sin \theta+1)\quad \text{con}\,\, \theta \in \left[\dfrac{\pi}{4},\dfrac{5}{4}\pi\right]. \end{equation*}

    Per calcolare \theta iniziale abbiamo impostato il seguente sistema

        \begin{equation*} \begin{cases} \sqrt{2}\cos\theta+1=2\\\\ \sqrt{2}\sin\theta+1=2 \end{cases} \quad \Leftrightarrow \quad \begin{cases} \cos \theta=\dfrac{\sqrt{2}}{2}\\\\ \sin\theta=\dfrac{\sqrt{2}}{2} \end{cases} \quad \Leftrightarrow \quad \theta=\dfrac{\pi}{4}=\theta_i \end{equation*}

    e per \theta finale

        \begin{equation*} \begin{cases} \sqrt{2}\cos\theta+1=0\\\\ \sqrt{2}\sin\theta+1=0 \end{cases} \quad \Leftrightarrow \quad \begin{cases} \cos \theta =-\dfrac{\sqrt{2}}{2}\\\\ \sin \theta=-\dfrac{\sqrt{2}}{2} \end{cases} \quad \Leftrightarrow \quad \theta=\dfrac{5}{4}\pi=\theta_f. \end{equation*}

    Calcoliamo8

        \[&\int_{\gamma_2} \omega_2 = \int_{\frac{\pi}{4}}^{\frac{5\pi}{4}} (\sqrt{2}\sin \theta+1, (\sqrt{2}\cos \theta+1)(\sqrt{2}\sin \theta+1)) \cdot (-\sqrt{2} \sin \theta, \sqrt{2} \cos \theta) \; d \theta =\]

        \[&=\int_{\frac{\pi}{4}}^{\frac{5}{4}\pi}\left(-2\sin^2\theta-\sqrt{2}\sin\theta+\sqrt{2}\cos\theta(\sqrt{2}\cos \theta+1)(\sqrt{2}\sin \theta+1)\right)\, d \theta=\]

        \[&=\int_{\frac{\pi}{4}}^{\frac{5}{4}\pi}\left(-2\sin^2\theta-\sqrt{2}\sin\theta+2\sqrt{2}\cos^2\theta\sin\theta+2\cos^2\theta+2\cos\theta\sin\theta+\sqrt{2}\cos\theta\right)\,d \theta=\]

        \[&=\int_{\frac{\pi}{4}}^{\frac{5}{4}\pi}\left(2\left(\cos^2\theta-\sin^2\theta\right)+\sqrt{2}\left(\cos\theta-\sin\theta\right)+2\sqrt{2}\cos^2\theta\sin\theta+2\cos\theta\sin\theta\right)\, d \theta=\]

        \[&=2\int_{\frac{\pi}{4}}^{\frac{5}{4}\pi}\cos\left(2\theta\right)\, d \theta+2\int_{\frac{\pi}{4}}^{\frac{5}{4}\pi}\cos\left(\theta+\dfrac{\pi}{4}\right)\, d \theta+\]

        \[&+2\sqrt{2}\int_{\frac{\pi}{4}}^{\frac{5}{4}\pi}\cos^2\theta\sin \theta\, d \theta+2 \int_{\frac{\pi}{4}}^{\frac{5}{4}\pi}\cos\theta\sin \theta \, d \theta=\]

        \[&=\sin\left(2\theta\right)\bigg \vert^{\frac{5}{4}\pi}_{\frac{\pi}{4}}+2\sin \left(\theta+\dfrac{\pi}{4}\right)\bigg \vert^{\frac{5}{4}\pi}_{\frac{\pi}{4}}-\dfrac{2\sqrt{2}}{3}\cos^3\theta \bigg \vert^{\frac{5}{4}\pi}_{\frac{\pi}{4}}+\sin^2\theta\bigg \vert^{\frac{5}{4}\pi}_{\frac{\pi}{4}}=\]

        \[&=\sin\left(\dfrac{5}{2}\pi\right)-\sin\left(\dfrac{\pi}{2}\right)+2\sin\left(\dfrac{5}{4}\pi+\dfrac{\pi}{4}\right)-2\sin\left(\dfrac{\pi}{4}+\dfrac{\pi}{4}\right)-\dfrac{2}{3}\sqrt{2}\cos^3\left(\dfrac{5}{4}\pi\right)+\]

        \[&\quad +\dfrac{2}{3}\sqrt{2}\sin^3\left(\dfrac{\pi}{4}\right) +\sin^2\left(\dfrac{5}{4}\pi\right)-\sin^2\left(\dfrac{\pi}{4}\right)=\]

        \[&=1-1+2\sin\left(\dfrac{3}{2}\pi\right)-2\sin\left(\dfrac{\pi}{2}\right)+\dfrac{2}{3}\sqrt{2}\left(\dfrac{\sqrt{2}}{2}\right)^3+\dfrac{2}{3}\sqrt{2}\left(\dfrac{\sqrt{2}}{2}\right)^3+\left(\dfrac{\sqrt{2}}{2}\right)^2-\left(\dfrac{\sqrt{2}}{2}\right)^2=\]

        \[&=-2-2+\dfrac{2}{3}\sqrt{2}\left(\dfrac{2\sqrt{2}}{8}+\dfrac{2\sqrt{2}}{8}\right) =-4+\dfrac{2}{3}\sqrt{2}\left(\dfrac{\sqrt{2}}{2}\right)=-4+\dfrac{2}{3}= - \dfrac{10}{3}. \end{aligned}\]

    Infine non resta che calcolare la parametrizzazione del sostegno di \gamma_3

        \begin{equation*} r_3(t) = (t,0) \quad \text{con}\,\,t\in \left[0,2\right] \end{equation*}

    da cui9

        \begin{equation*} \int_{\gamma_3} \omega_2 = 0. \end{equation*}

    Infine

        \[\,\]

        \[\int_\gamma \omega=\dfrac{9}{2}-\dfrac{10}{3}+\dfrac{23}{6} = 5.\]

        \[\,\]

    Era altresì possibile calcolare \int_{\gamma}\omega_2 applicando il Teorema di Gauss-Green. Consideriamo il percorso chiuso in figura

        \[\,\]

        \[\,\]

        \[\,\]

    Rendered by QuickLaTeX.com

        \[\,\]

        \[\,\]

        \[\,\]

    dove il sostegno di \gamma_2 è la semicirconferenza, quello di \gamma_4 è il diametro della semicirconferenza, quello di \gamma_5 è il segmento che congiunge i punti (0,0) e (1,1) ed infine \gamma_3 è il segmento che congiunge i punti (0,0) e (2,0). Dunque

        \begin{equation*} \int_{\gamma_2} \omega_2 + \int_{\gamma_4} \omega_2 = \iint_{\Omega} (y-1) \, dx \, dy \end{equation*}

    dove \Omega è il dominio contenuto nel percorso chiuso. Calcoliamo10

        \begin{align*} \iint_{\Omega} (y-1) \, dx \, dy & = \int_{\frac{\pi}{4}}^{\frac{5\pi}{4}} d\theta \int_0^{\sqrt{2}} \rho (\rho \sin \theta + 1 - 1) \, d\rho = \\ & = \int_{\frac{\pi}{4}}^{\frac{5\pi}{4}} d\theta \int_0^{\sqrt{2}} \rho^2 \sin \theta \; d\rho = \\ &=\int_{\frac{\pi}{4}}^{\frac{5\pi}{4}}\sin \theta\, d \theta \int_0^{\sqrt{2}} \rho^2 \, d \rho= \dfrac{4}{3}. \end{align*}

    Ora parametrizziamo il sostegno di \gamma_5

        \begin{equation*} r_5(t)=(t,t) \quad \text{con}\,\, t \in \left[0,1\right] \end{equation*}

    allora

        \begin{equation*} \int_{\gamma_5} \omega_2 = \int_0^1 (t,t^2)\cdot(1,1) \, dt = \dfrac{5}{6}. \end{equation*}

    Quindi

        \begin{align*} \int_\gamma\omega&=\int_\gamma\omega_1+\int_\gamma\omega_2=\dfrac{9}{2}+\int_\gamma\omega_2=\\ &=\dfrac{9}{2}+\iint_{\Omega}(y-1)\,dxdy-\int_{\gamma_5}\omega_2+\int_{\gamma_3}\omega_2=\\ &=\dfrac{9}{2}+\dfrac{4}{3}-\dfrac{5}{6}+0=5. \end{align*}

    Si conclude nuovamente che

        \[\,\]

        \[\int_{\gamma}\omega=5.\]

        \[\,\]

    Esempio 20.  Calcolare \int_{\gamma} \omega dove

        \begin{equation*} \omega = \dfrac{y}{(x-1)^2+y^2} \, dx - \dfrac{2y}{x^2+y^2} \, dx + \dfrac{2x}{x^2+y^2} \, dy + \dfrac{1-x}{(x-1)^2+y^2} \, dy \end{equation*}

    e il sostegno di \gamma è rappresentato dalla circonfenreza di equazione x^2+y^2=9, percorsa in senso antiorario.

        \[\,\]

    Soluzione.

        \[\,\]

    Riscriviamo \omega come segue

        \begin{equation*} \omega=\underbrace{\dfrac{y}{(x-1)^2+y^2} \, dx+ \dfrac{1-x}{(x-1)^2+y^2} \, dy }_{\omega_1} +\underbrace{- \dfrac{2y}{x^2+y^2}dx+\dfrac{2x}{x^2+y^2} \, dy}_{\omega_2} \end{equation*}

    Vogliamo verificare se \omega_1 è chiusa e a tal scopo calcoliamo le derivate miste. Poniamo

        \begin{equation*} F_1 = \dfrac{y}{(x-1)^2+y^2} \end{equation*}

    e deriviamola rispetto ad y

        \begin{equation*} \dfrac{\partial F_1}{\partial y} =\dfrac{(x-1)^2+y^2-y(2y)}{\left(\left(x-1\right)^2+y^2\right)^2}= \dfrac{x^2+1-2x-y^2}{\left((x-1)^2+y^2\right)^2}. \end{equation*}

    Definiamo poi

        \begin{equation*} F_2 = \dfrac{1-x}{(x-1)^2+y^2} \end{equation*}

    e deriviamola rispetto ad x

        \begin{equation*} \dfrac{\partial F_2}{\partial x} =\dfrac{\left(-1\right)\left(\left(x-1\right)^2+y^2\right)+2\left(x-1\right)^2}{\left((x-1)^2+y^2\right)^2}= \dfrac{x^2+1-2x-y^2}{\left((x-1)^2+y^2\right)^2}. \end{equation*}

    Osserviamo che

        \begin{equation*} \dfrac{\partial F_1}{\partial y}=\dfrac{\partial F_2}{\partial x} \end{equation*}

    quindi \omega_1 è chiusa. Si osserva che \text{Dom}\left(\omega_1\right) = \mathbb{R}^2 \setminus \{(1,0) \} che non è un insieme semplicemente connesso quindi non abbiamo informazioni al momento per dire se \omega_1 è esatta nel suo dominio. Consideriamo il percorso chiuso in figura 1

        \[\,\]

        \[\,\]

        \[\,\]

    Rendered by QuickLaTeX.com

        \[\,\]

        \[\,\]

        \[\,\]

    dove il sostegno di \gamma è la circonferenza centrata nell’origine di raggio 3, il sostegno di \gamma_1 è una circonferenza centrata in (1,0) di raggio 1, \gamma_2 è il segmento che congiunge i punti (0,-3) e (0,0). Sostanzialmente abbiamo scelto un percorso chiuso che permette di poter applicare il Teorema di Gauss-Green; inoltre il dominio contenuto all’interno di tale percorso è un insieme semplicemente connesso quindi \omega_1 è localmente esatta. Applicando il Teorema di Gauss-Green abbiamo

    (25)   \begin{equation*} \begin{aligned} \int_{+\gamma}\omega_1+&\int_{-\gamma_1}\omega_1+\underbrace{\int_{+\gamma_2}\omega_1+\int_{-\gamma_2}\omega_1}_{=0}=\\ &=\iint_A\left( \dfrac{\partial F_2}{\partial x}(x,y)-\dfrac{\partial F_1}{\partial y}(x,y)\right) \,dxdy=\iint_A\left( 0\right) \,dxdy=0, \end{aligned} \end{equation*}

    dove

        \begin{equation*} A=\{(x,y)\in \mathbb{R}^2:\,x^2+y^2\leq9,\, (x-1)^2+y^2\geq 1\}, \end{equation*}

    da cui

    (26)   \begin{equation*} \int_{+\gamma}\omega_1 = -\int_{-\gamma_1}\omega_1 = \int_{+\gamma_1}\omega_1. \end{equation*}

    La parametrizzazione di \gamma_1 è

        \begin{equation*} r_1(\theta)= \left(\cos \theta + 1, \sin \theta\right), \qquad \mbox{con } \theta \in [0,2\pi]. \end{equation*}

    Calcoliamo la derivata di \gamma_1(\theta)

        \begin{equation*} r^\prime_1(\theta)=(-\sin \theta,\cos \theta). \end{equation*}

    Dal momento che

        \begin{equation*} \begin{cases} dx=-\sin \theta \, d \theta\\\\ dy=\cos \theta \, d \theta \end{cases} \end{equation*}

    restringendo \omega_1 lungo r_1(\theta) si ha

        \begin{align*} \omega_1\left(\gamma_1(\theta)\right)&=\dfrac{\sin \theta \left(-\sin \theta\right)}{\left(\cos \theta +1-1\right)^2+\sin^2 \theta}\, d \theta +\dfrac{\left(1-\left(\cos \theta +1\right)\right)\left(\cos \theta\right)}{\left(\cos \theta +1-1\right)^2+\sin^2 \theta} \, d \theta=\\ &=-\dfrac{\sin^2 \theta }{\cos^2 \theta+\sin^2\theta}\, d \theta-\dfrac{\cos^2 \theta}{\cos^2 \theta+\sin^2 \theta}\, d \theta=\\ &=-\left(\sin^2 \theta +\cos^2 \theta\right)\, d \theta=-d \theta, \end{align*}

    da cui tornando a 26 si ha

        \begin{equation*} \int_{+\gamma}\omega_1 = \int_{+\gamma_1}\omega_1=-\int_{0}^{2\pi}d \theta=-2 \pi. \end{equation*}

    Non resta che calcolare \int_{\gamma}\omega_2. Vogliamo verificare se \omega_2 sia chiusa e a tal scopo calcoliamo le derivate miste. Poniamo

        \[F_3 = - \dfrac{2y}{x^2+y^2},\]

    e derivandolo rispetto ad y otteniamo

        \[\dfrac{\partial F_3}{\partial y} = - \dfrac{2(x^2+y^2)-2y(2y)}{(x^2+y^2)^2} = \dfrac{-2x^2+2y^2}{(x^2+y^2)^2}.\]

    Poniamo poi

        \[F_4 = \dfrac{2x}{x^2+y^2}\]

    e derivando anche quest’ultima stavolta rispetto ad x otteniamo

        \[\dfrac{\partial F_4}{\partial x} = - \dfrac{2(x^2+y^2)-2x(2x)}{(x^2+y^2)^2} = \dfrac{-2x^2+2y^2}{(x^2+y^2)^2}.\]

    Osserviamo che

        \[\dfrac{\partial F_3}{\partial y} =\dfrac{\partial F_4}{\partial x}\]

    quindi \omega_2 è chiusa. Si osserva che \text{Dom}\left(\omega_2\right) = \mathbb{R}^2 \setminus \{(0,0) \} che non è un insieme semplicemente connesso quindi non abbiamo informazioni al momento per dire se \omega_2 è esatta nel suo dominio. Consideriamo il percorso chiuso rappresentato in Figura 2

        \[\,\]

        \[\,\]

        \[\,\]

    Rendered by QuickLaTeX.com

        \[\,\]

        \[\,\]

        \[\,\]

    dove il sostegno di \gamma è la circonferenza centrata nell’origine di raggio 3, il sostegno di \gamma_3 è una circonferenza centrata in (0,0) di raggio 1 e \gamma_4 è il segmento che congiunge i punti (-3,0) e (0,0). Il dominio contenuto all’interno del nostro percorso chiuso è un insieme semplicemente connesso quindi \omega_2 è localmente esatta. Procedendo in modo analogo al passaggio 25 si ottiene

    (27)   \begin{equation*} \int_{+\gamma}\omega_2 = \int_{+ \gamma_3}\omega_2. \end{equation*}

    Parametrizziamo \gamma_3(\theta):

        \begin{equation*} r_3(\theta) = (\cos \theta, \sin \theta), \qquad \mbox{con } \theta \in [0,2\pi] \end{equation*}

    e calcoliamo la derivata di \gamma_3(\theta)

        \begin{equation*} r^\prime_3(\theta)=\left(-\sin \theta,\cos \theta \right). \end{equation*}

    Dal momento che

        \begin{equation*} \begin{cases} dx=-\sin \theta\, d \theta \\ dy=\cos\theta\, d \theta \end{cases} \end{equation*}

    restringendo \omega_2 lungo r_3(\theta)

        \begin{align*} \omega_2\left(r_3\left(\theta\right)\right) & = -\dfrac{2 \sin \theta(-\sin \theta)}{\cos^2\theta+\sin^2\theta}\,d \theta+\dfrac{2\cos \theta(\cos \theta)}{\cos^2 \theta+\sin^2 \theta} \, d\theta=\\ &=2 \left(\sin^2\theta+\cos^2 \theta\right)\, d \theta=2 \, \theta. \end{align*}

    Tornando a 27 otteniamo

        \begin{equation*} \int_{+\gamma}\omega_2 = \int_{+ \gamma_3}\omega_2= 2 \int_0^{2\pi} d\theta = 4\pi \end{equation*}

    e quindi concludiamo che

        \begin{equation*} \int_{+\gamma} \omega =\int_{+\gamma} \omega_1+\int_{+\gamma} \omega_2= -2\pi+4\pi = 2\pi \end{equation*}

    pertanto

        \[\,\]

     

        \[\int_\gamma \omega= 2\pi.\]

        \[\,\]

    Esempio 21.  Calcolare l’integrale curvilineo:

        \begin{equation*} \int_\gamma y(1-y^2+\cos(x)) \; dx+(x(1+x^2)+\sin (x)+y) \; dy \end{equation*}

    Dove \gamma è la poligonale non chiusa che unisce i seguenti punti nell’ordine asse\-gnato: (0,0), (2,0), (1,2) e (0,2).

        \[\,\]

    Soluzione.

        \[\,\]

    Il grafico del sostegno di \gamma è riportato nella figura seguente

        \[\,\]

    Rendered by QuickLaTeX.com

        \[\,\]

    Poniamo

        \begin{align*} & F_1=(y-y^3+y\cos x)\\ & F_2=(x+x^3+\sin x+y) \end{align*}

    da cui:

        \begin{align*} \int_\gamma \omega &= \int_\gamma (y-y^3+y\cos x) \; dx+(x+x^3+\sin x+y)\; dy=\\ &=\int_\gamma F_1 \; dx+F_2 \; dy. \end{align*}

    Osserviamo che

        \begin{align*} &\dfrac{\partial F_2}{\partial x}=1+3x^2+\cos x\\\\ &\dfrac{\partial F_1}{\partial y}=1-3y^2+\cos x \end{align*}

    e quindi

        \begin{equation*} \dfrac{\partial F_2}{\partial x} \neq \dfrac{\partial F_1}{\partial y} \end{equation*}

    pertanto \omega non è chiusa. Sia \omega_1=(y+y\cos x)\;dx+(x+\sin x+y) \; dy che è chiusa in quanto

        \begin{equation*} \dfrac{\partial \left(x+\sin x+y\right)}{\partial x}=\dfrac{\partial \left(y+y\cos x\right)}{\partial y}=\cos x +1. \end{equation*}

    Inoltre il suo dominio è \mathbb{R}^2 che è un insieme semplicemente connesso e pertanto risulta esatta. Calcoliamo un potenziale U(x,y) per \omega_1 che rispetta le seguente condizioni:

        \begin{equation*} \begin{cases} \dfrac{\partial U}{\partial x}(x,y)=y+y\cos x\\\\ \dfrac{\partial U}{\partial y}(x,y)=x+\sin x+y \end{cases}. \end{equation*}

    Dalla prima equazione ricaviamo che

        \begin{equation*} U(x,y)=yx+y\sin x+c(y) \end{equation*}

    sostituiamo U(x,y) nella seconda equazione:

        \begin{equation*} x+\sin x+\dfrac{dc(y)}{dy}=x+\sin x+y \Leftrightarrow c(y)=\dfrac{y^2}{2}+\mbox{costante} \end{equation*}

    da cui

        \begin{equation*} U(x,y)=yx+y\sin x+\dfrac{y^2}{2}+\mbox{costante}. \end{equation*}

    Si conclude che

        \begin{equation*} \int_\gamma \omega_1=U(0,2)-U(0,0)=\dfrac{4}{2}=2 \end{equation*}

    Sia ora

        \begin{equation*} \omega_2=-y^3 \; dx+x^3 \; dy. \end{equation*}

    Applichiamo il Teorema di Gauss-Green:

        \begin{equation*} \int_\gamma \omega_2=\iint_D (3x^2+3y^2)dxdy-\int_{\gamma_1} \omega_2 \end{equation*}

    dove \gamma_1 è il sostegno rappresentato dal segmento che va dal punto (0,2) a (0,0). Abbiamo dunque

        \begin{align*} &D= \left\{ \left(x,y\right)\in \mathbb{R}^2 \; \bigg\vert \; 0 \le y \le 2, \, 0 \le x \le \dfrac{4-y}{2} \right\}\\ &\gamma_1:\,r_1(t)=(0,-t), \qquad \mbox{con } t \in (-2,0) \end{align*}

    e quindi

        \begin{align*} \int_{\gamma} \omega_2 & =\iint_D (3x^2+3y^2)dxdy-\int_{\gamma_1} \omega_2 \\ & = 3\int_0^2 \; dy \int_0^{\frac{4-y}{2}}(x^2+y^2) \; dx+ \int_{-2}^0 (t^3,0) \cdot (0,-1) \; dt =\\ &=3 \int_0^2 \left(\dfrac{x^3}{3}+y^2x \right)\bigg\vert_0^{\frac{4-y}{2}} \; dy =\\ & = 3 \int_0^2 \left( \dfrac{1}{3} \left( \dfrac{4-y}{2} \right)^3 +y^2 \left(\dfrac{4-y}{2} \right) \right)dy=\dfrac{35}{2}. \end{align*}

    Concludiamo che

        \[\int_\gamma \omega=\dfrac{35}{2}+2=\dfrac{35+4}{2}=\dfrac{39}{2}.\]

        \[\,\]

    Esempio 22.  Calcolare

        \begin{equation*} \int_{\gamma} \left(\dfrac{1}{\sqrt{x^2+1}} \; \arctan \dfrac{y}{\sqrt{x^2+1}}\right) \, dx + \left(2x + \dfrac{1}{\sqrt{y^2+1} \; }\arctan\dfrac{x}{\sqrt{y^2+1}}\right) \, dy \end{equation*}

    dove \gamma è il sostegno della curva di equazione \displaystyle\frac{x^2}{16}+\frac{y^2}{9}=1 percorsa una volta in senso antiorario.

        \[\,\]

    Soluzione.

        \[\,\]

    Il dominio di \omega è \mathbb{R}^2: definiamo

        \begin{equation*} F_1 = \dfrac{1}{\sqrt{x^2+1}} \; \arctan \dfrac{y}{\sqrt{x^2+1}} \end{equation*}

    e

        \begin{equation*} F_2 =2x + \dfrac{1}{\sqrt{y^2+1} \; }\arctan\dfrac{x}{\sqrt{y^2+1}}. \end{equation*}

    Vogliamo studiare la chiusura di \omega, quindi calcoliamo le derivate miste:

        \begin{equation*} \dfrac{\partial F_1}{\partial y} = \dfrac{1}{x^2+y^2+1} \qquad \mbox{e} \qquad \dfrac{\partial F_2}{\partial x} = 2+\dfrac{1}{x^2+y^2+1} \end{equation*}

    e osserviamo che

        \begin{equation*} \dfrac{\partial F_1}{\partial y} \neq \dfrac{\partial F_2}{\partial x}, \end{equation*}

    pertanto \omega non è chiusa. Applichiamo dunque il Teorema di Gauss-Green

        \begin{equation*} \int_\gamma \omega = \iint_C \left(\dfrac{\partial F_2}{\partial y}-\dfrac{\partial F_1}{\partial x} \right) \, dx \, dy \end{equation*}

    dove

        \begin{equation*} C = \left\{(x,y) \in \mathbb{R}^2 \, \bigg\vert \, \frac{x^2}{16}+\frac{y^2}{9}\leq 1 \right\} \end{equation*}

    è rappresentato nella figura alla pagina seguente. Dunque

        \begin{align*} \int_{\gamma} \omega & = - \iint_{C}\left( -\dfrac{1}{x^2+y^2+1} + 2 + \dfrac{1}{x^2+y^2+1}\right)\; dx \; dy =\\\\ & = \iint_{C} 2 \; dx \; dy \end{align*}

        \[\,\]

        \[\,\]

        \[\,\]

    Rendered by QuickLaTeX.com

        \[\,\]

        \[\,\]

        \[\,\]

    Infine si ricorda che data un’ellisse di equazione cartesiana:

        \begin{equation*} \dfrac{x^2}{a^2}+\dfrac{y^2}{b^2}=1 \quad \text{con}\,\, a,b >0, \end{equation*}

    la sua area è pari a

        \begin{equation*} A=\iint_{\frac{x^2}{a^2}+\frac{y^2}{b^2}\leq 1 }1\,dxdy=\pi a b . \end{equation*}

    Ritornando all’integrale abbiamo

        \begin{equation*} \int_{\gamma} \omega=\iint_{C} 2 \; dx \; dy=2\left(12\pi \right)=24\pi, \end{equation*}

    da cui

        \[\,\]

        \[\int_{\gamma}\omega=24\pi.\]

        \[\,\]

        \[\,\]


    1. L’equazione che descrive il sostegno è \left(x-1\right)^2+\left(y-1\right)^2=\left(\frac{1}{2}\sqrt{2^2+2^2}\right)^2=2. Quindi il raggio è r=\sqrt{2} da cui si trova facilmente la parametrizzazione \bf{r}_2(\theta)=\left(\sqrt{2}\cos\theta+1,\sqrt{2}\sin\theta+1\right).
    1. Si ricorda che \cos\theta-\sin\theta=\sqrt{2}\cos\left(\theta+\dfrac{\pi}{4}\right) e \cos\left(2\theta\right)=\cos^2\theta-\sin^2\theta.
    1. L’integrale di linea è nullo perché \omega_2 ristretta su r_3(t) è identicamente nulla.
    1. La parametrizzazione della semicerchio è

          \[\begin{cases} x=\rho\cos \theta+1\\ \quad \quad \quad \quad \quad \quad\quad\quad \theta\in \left[\dfrac{\pi}{4},\dfrac{5}{4}\pi\right]\rho \in \left[0,\sqrt{2}\right]\\ y=\rho \sin \theta. \end{cases}\]

      Inoltre si ricorda che

          \[dx\,dy=\rho \, d\rho d \theta.\]

        \[\,\]

        \[\,\]

    Convenzioni verso di percorrenza

        \[\,\]

        \[\,\]

    Leggi...

    In molte applicazioni risulta necessario calcolare un integrale di linea di una forma differenziale su un bordo costituito da una parte esterna e una interna (ad esempio una corona circolare): in tal caso per convenzione si sceglie come verso di percorrenza quello antiorario per la parte esterna ed orario per la parte interna. La motivazione è legata alla scelta della terna di riferimento locale che deve essere così fatta: versore normale \hat{n} uscente (rispetto alla regione di interesse), versore ortogonale \hat{z} al piano del foglio uscente, conseguentemente il versore tangente \hat{t}=\hat{n}\wedge\hat{z} risulta diretto in senso antiorario nella parte esterna mentre si avvolge in senso orario nella parte interna (seguendo la regola della mano destra). Si può immaginare per comodità una persona che cammina lungo tali bordi: per essa il dominio deve rimanere sempre a sinistra e pertanto egli percorrerà il bordo esterno in senso antiorario e quello interno in senso orario.

        \[\,\]

    Esempio 23.  Calcolare il seguente integrale di seconda specie

    (28)   \begin{equation*} I=\int_{+\gamma}\left(-y+2x\right)\,dx+\left(x+1\right)\, dy=\int_{+\gamma}F_1\,dx+F_2\,dy=\int_{+\gamma}\omega \end{equation*}

    dove il sostegno di \gamma è dato dalla frontiera del dominio tratteggiato in figura (regolare, a 4 contorni), con le circonferenze non aventi il centro nell’origine che hanno raggio 1/2 e centri rispettivamente (-2,0) e (2,0). Nota. Si assuma la curva orientata come in figura.

        \[\,\]

        \[\,\]

        \[\,\]

    Rendered by QuickLaTeX.com

        \[\,\]

        \[\,\]

        \[\,\]

    Soluzione.

        \[\,\]

    Riscriviamo 28 come segue

        \begin{equation*} I=\int_{\gamma}\underbrace{y\,dy+\left(x+1\right)\,dx}_{\omega_1}+2\int_{\gamma}\underbrace{\left(x-y\right)\,dx}_{\omega_2}. \end{equation*}

    Si osserva facilmente che \omega_1 è chiusa e inoltre il suo dominio è \mathbb{R}^2 che è un insieme semplicemente connesso: pertanto l’integrale di linea di seconda specie sul percorso chiuso \gamma risulta nullo \int_{\gamma_1}\omega=0 . Definiamo \gamma=\gamma_1\cup\gamma_2\cup\gamma_3\cup\gamma_4 come in figura

        \[\,\]

        \[\,\]

        \[\,\]

    Rendered by QuickLaTeX.com

        \[\,\]

        \[\,\]

        \[\,\]

    dove \gamma_1 ha come sostegno una circonferenza di equazione x^2+y^2=1, mentre \gamma_2 una circonferenza di equazione (x-2)^2+y^2=1/4, \gamma_3 una circonferenza di equazione (x+2)^2+y^2=1/4 ed infine \gamma_4 una circonferenza di equazione x^2+y^2=9.\\ Per quanto detto, possiamo riscrivere 28 come segue

    (29)   \begin{equation*} I=\int_{-\gamma_1}\omega_2+\int_{-\gamma_2}\omega_2+\int_{-\gamma_3}\omega_2+\int_{+\gamma_4}\omega_2. \end{equation*}

    Parametrizziamo il sostegno di \gamma_1

        \begin{equation*} r_1\left(\theta\right)=\left(\cos \theta,-\sin \theta\right) \quad \text{con}\,\, \theta \in [0,2\pi] \end{equation*}

    e derivando otteniamo

        \begin{equation*} r_1^\prime\left(\theta\right)=\left(-\sin \theta,-\cos \theta \right) \end{equation*}

    per cui

    (30)   \begin{equation*} \int_{-\gamma_1} \omega_2=\int_{0}^{2\pi}\left(\cos\theta+\sin\theta\right)\left(-\sin\theta\right)\,d \theta. \end{equation*}

    Parametrizziamo il sostegno di \gamma_2 come segue

        \begin{equation*} r_2\left(\theta\right)=\left(2+\dfrac{1}{2}\cos \theta,-\dfrac{1}{2}\sin \theta\right)\quad \text{con}\,\, \theta\in\left[0,2\pi\right] \end{equation*}

    e derivando quanto ottenuto abbiamo

        \begin{equation*} r_2\left(\theta\right)=\left(-\dfrac{1}{2}\sin\theta,-\dfrac{1}{2}\cos\theta\right), \end{equation*}

    per cui

    (31)   \begin{equation*} \int_{-\gamma_2}\omega_2=\int_{0}^{2\pi}\left(2+\dfrac{1}{2}\cos \theta+\dfrac{1}{2}\sin \theta\right)\left(-\dfrac{1}{2}\sin \theta\right)\, d\theta. \end{equation*}

    Parametrizziamo ora il sostegno di \gamma_3

        \begin{equation*} r_3\left(\theta\right)=\left(-2+\dfrac{1}{2}\cos\theta,-\dfrac{1}{2}\sin\theta\right) \quad \text{con}\,\, \theta \in \left[0,2\pi\right] \end{equation*}

    e la sua derivata è

        \begin{equation*} r_3^\prime\left(\theta\right)=\left(-\dfrac{1}{2}\sin\theta,-\dfrac{1}{2}\cos\theta\right) \end{equation*}

    per cui

    (32)   \begin{equation*} \int_{-\gamma_3}\omega_2=\int_{0}^{2\pi}\left(-2+\dfrac{1}{2}\cos\theta+\dfrac{1}{2}\sin\theta\right)\left(-\dfrac{1}{2}\sin\theta\right)\,d \theta. \end{equation*}

    Infine parametrizziamo il sostegno di \gamma_4

        \begin{equation*} r_4\left(\theta\right)=\left(3\cos \theta,3\sin \theta \right) \quad \text{con}\,\, \theta \in \left[0,2\pi\right] \end{equation*}

    e calcoliamo la sua derivata

        \begin{equation*} r^\prime_4\left(\theta\right)=\left(-3\sin \theta,3\cos \theta\right) \end{equation*}

    per cui

    (33)   \begin{equation*} \int_{+\gamma_4}\omega_2=\int_{0}^{2\pi}\left(3\cos\theta-3\sin\theta\right)\left(-3\sin\theta\right)\,d \theta. \end{equation*}

    Pertanto usando 30,31,32 e 33 possiamo riscrivere 29 come segue

        \begin{align*} I&=\int_{0}^{2\pi}\left(\cos\theta+\sin\theta\right)\left(-\sin\theta\right)\,d \theta+\\ &+\int_{0}^{2\pi}\left(2+\dfrac{1}{2}\cos \theta+\dfrac{1}{2}\sin \theta\right)\left(-\dfrac{1}{2}\sin \theta\right)\, d\theta+\\ &+\int_{0}^{2\pi}\left(-2+\dfrac{1}{2}\cos\theta+\dfrac{1}{2}\sin\theta\right)\left(-\dfrac{1}{2}\sin\theta\right)\,d \theta+\\ &+\int_{0}^{2\pi}\left(3\cos\theta-3\sin\theta\right)\left(-3\sin\theta\right)\,d \theta=\\ &\int_{0}^{2\pi}(\cos\theta+\sin\theta+1+\dfrac{1}{4}\cos\theta+\dfrac{1}{4}\sin\theta-1+\dfrac{1}{4}\cos\theta+\dfrac{1}{4}\sin\theta+\\ &9\cos\theta-9\sin\theta)\left(-\sin\theta\right)\,d \theta=\\ &=\int_{0}^{2\pi}\left(\dfrac{21}{2}\cos\theta-\dfrac{15}{2}\sin\theta\right)\left(-\sin\theta\right)\,d \theta=\\ &=\dfrac{21}{2}\int_{0}^{2\pi}\cos\left(\theta\right)\left(-\sin\theta\right)\,d \theta+\dfrac{15}{2}\int_{0}^{2\pi}\sin^2\theta\,d \theta\overset{\clubsuit}{=}\\ &=\dfrac{\cos^2\theta}{2}\bigg\vert^{2\pi}_0+\dfrac{15}{2}\int_{0}^{2\pi}\dfrac{1-\cos\left(2\theta\right)}{2}\,d \theta=\\ &=\dfrac{\cos^2\left(2\pi\right)}{2} -\dfrac{\cos^2\left(0\right)}{2}+\dfrac{15}{2}\int_{0}^{2\pi}\dfrac{1}{2}\,d \theta-\dfrac{15}{2}\int_{0}^{2\pi}\dfrac{\cos\left(2\theta\right)}{2}\,d \theta\overset{\diamond}{=}\\ &=\dfrac{1}{2}-\dfrac{1}{2}+\dfrac{15}{4}\cdot\left(2\pi\right)+0=\dfrac{15}{2}\pi\\ \end{align*}

    dove in \clubsuit abbiamo usato \sin^2(\theta)=\frac{1-\cos(2\theta)}{2} e in \diamond abbiamo usato il fatto che l’integrale del coseno su un multiplo del suo periodo è nullo. Dunque possiamo riscrivere 29 come segue

        \begin{equation*} I=2\cdot \dfrac{15}{2}\pi=15\pi. \end{equation*}

    Si conclude che

        \[\,\]

        \[\boxcolorato{analisi}{ I=15\pi. }\]

        \[\,\]

        \[\,\]

    Esercizi di riepilogo

        \[\,\]

        \[\,\]

    Leggi...

    In questo paragrafo conclusivo proponiamo esercizi di varia natura riguardo l’integrazione delle forme e dei campi.

        \[\,\]

    Esempio 26.  Calcolare

    (34)   \begin{equation*} \int_{\gamma}\omega= \int_{\gamma} -\dfrac{3}{3x+y+2} \, dx + \dfrac{3x+y+1}{3x+y+2} \, dy \end{equation*}

    dove \gamma è la curva di punto iniziale A(0,-1) e punto finale B, costituita dai seguenti archi

  • il minore dei due archi AD della circonferenza di equazionex^2+y^2-4x+2y+1=0 delimitato dai punti A e D(2,1)
  • il segmento che unisce D al punto C(1,5)
  • l’arco {CB} della parabola di equazione y=-\frac{x^2}{2}+3x+\frac{5}{2} delimitato da C e dal suo simmetrico rispetto all’asse della parabola stessa.
  •     \[\,\]

    Soluzione.

        \[\,\]

    Rappresentiamo graficamente il sostegno di \gamma:

        \[\,\]

        \[\,\]

        \[\,\]

    Rendered by QuickLaTeX.com

        \[\,\]

        \[\,\]

        \[\,\]

    Osserviamo che il dominio di \omega è:

        \[D = \left\{(x,y) \in \mathbb{R}^2 \, \vert \, 3x+y+2 \neq 0\right\}\]

    e vogliamo studiare se \omega sia chiusa nel suo dominio. Chiamiamo

        \[F_1 = \dfrac{-3}{3x+y+2} \quad \mbox{e} \quad F_2 = \dfrac{3x+y+1}{3x+y+2},\]

    calcoliamo le derivate

        \[\dfrac{\partial F_1}{\partial y} = \dfrac{3}{(3x+y+2)^2} \quad \mbox{e} \quad \dfrac{\partial F_2}{\partial x} = \dfrac{3}{(3x+y+2)^2}\]

    e poiché

        \[\frac{\partial F_1}{\partial y} = \frac{\partial F _2}{\partial x}\]

    allora \omega risulta chiusa. Rappresentiamo D

        \[\,\]

        \[\,\]

        \[\,\]

    Rendered by QuickLaTeX.com

        \[\,\]

        \[\,\]

        \[\,\]

    e notiamo che D non è un insieme semplicemente connesso poiché vanno esclusi tutti i punti della retta di equazione 3x+y+2=0. Pertanto non possiamo dire nulla sull’esattezza di \omega in D. Tuttavia il percorso \gamma si trova in una zona del dominio che è un insieme semplicemente connesso (infatti la curva è tutta contenuta nel semipiano superiore rispetto alla retta) quindi \omega è localmente esatta. Possiamo calcolare un potenziale locale U tale che:

        \[\begin{cases} \dfrac{\partial U}{\partial x}= \dfrac{-3}{3x+y+2}\\\\ \dfrac{\partial U}{\partial y}= \dfrac{3x+y+1}{3x+y+2} \end{cases}\quad \Leftrightarrow \quad \begin{cases} U(x,y)=-\ln\left \vert 3x+y+2\right \vert + c(y)\\\\ \dfrac{\partial U}{\partial y}= \dfrac{3x+y+1}{3x+y+2} \end{cases}\]

    Sostituendo U(x,y) nella seconda equazione del sistema troviamo

        \[&\dfrac{\partial (-\ln\left( \vert 3x+y+2\right \vert + c(y))}{\partial y} = \dfrac{3x+y+1}{3x+y+2}=1-\dfrac{1}{3x+y+2}\quad \Leftrightarrow\]

        \[&\Leftrightarrow\quad -\dfrac{1}{3x+y+2}+\dfrac{dc}{dy}(y)=3x+y+2-\dfrac{1}{3x+y+2}\quad \Leftrightarrow\\ &\Leftrightarrow\quad \dfrac{dc}{dy}(y) = 1 \quad \Leftrightarrow\quad c(y)=y+costante,\]

    avendo utilizzato una proprietà11, quindi

        \[U(x,y,z)= - \ln\left \vert 3x+y+2\right \vert +y.\]

    Tornando a 34:

        \[U(B)-U(A) = \int_\gamma \omega\]

    con B=(5,5) e A=(0,-1), otteniamo

        \[U(B)-U(A) = -\ln(15+5+2)+5-(-\ln(0-1+2)-1) = 6 - \ln(22).\]

    Si conclude quindi

        \[\,\]

    boxcoloato

        \[\int_{\gamma}\omega=6 - \ln(22).\]

        \[\,\]

    Esempio 27.  Calcolare

        \[\int_{\gamma} \left(\dfrac{xy^3}{3} + \dfrac{y}{\sqrt{x^2+1}}-\sin y\right) \, dx + \left(x^2y^2+\operatorname {arcsinh} (x) - x \cos y\right) \, dy\]

    dove \gamma è la frontiera del rettangolo R=[1,4] \times [1,3] percorsa una volta in senso orario.

        \[\,\]

    < Soluzione.

        \[\,\]

    Osserviamo che il dominio di \omega è \mathbb{R}^2. Chiamiamo

        \[F_1 = \dfrac{xy^3}{3} + \dfrac{y}{\sqrt{x^2+1}}-\sin y \qquad \mbox{e} \qquad F_2 = x^2y^2+\operatorname {arcsinh} (x) - x \cos y\]

    e verifichiamo se \omega è chiusa nel suo dominio: calcoliamo le derivate miste

        \[\dfrac{\partial F_1}{\partial y} = xy^2+\dfrac{1}{\sqrt{x^2+1}}-\cos y \qquad \mbox{e} \qquad \dfrac{\partial F_2}{\partial x} = 2xy^2+\dfrac{1}{\sqrt{x^2+1}}-\cos y\]

    ed essendo

        \[\dfrac{\partial F_1}{\partial y} \neq \dfrac{\partial F_2}{\partial x}\]

    allora \omega non è chiusa nel suo dominio. Applichiamo ora il Teorema di Gauss-Green

        \[\int_\gamma \omega = \iint_R \left(\dfrac{\partial F_2}{\partial y}-\dfrac{\partial F_1}{\partial x}\right) \, dx \, dy\]

    dove

        \[R= [1,4] \times [1,3]\]

    ed è rappresentato in figura, percorso in senso orario:

        \[\,\]

        \[\,\]

        \[\,\]

    Rendered by QuickLaTeX.com

        \[\,\]

        \[\,\]

        \[\,\]

    Dunque

        \[\int_{\gamma}\omega &=\]

        \[&=-\iint_R\left( 2xy^2+\dfrac{1}{\sqrt{x^2+1}}-\cos y-\left(xy^2+\dfrac{1}{\sqrt{x^2+1}}-\cos y \right) \right) \,dxdy=\]

        \[&=-\iint_R xy^2\,dxdy\]

    dove si è usato il segno meno in quanto la curva è percorsa in senso opposto a quello richiesto nel teorema. Abbiamo dunque

        \[\int_{\gamma}\omega&=-\iint_Rxy^2\,dxdy= -\int_1^4 x \, dx \int_1^3 y^2 \; dy =\]

        \[&= -\dfrac{x^2}{2} \bigg\vert_1^4 \;\; \dfrac{y^3}{3} \bigg\vert_1^3 =- \dfrac{1}{2} (16-1) \cdot \dfrac{1}{3} (27-1) = -65.\]

    Si conclude che

        \[\,\]

        \[\boxcolorato{analisi}{\int_{\gamma}\omega=-65.}\]

        \[\,\]

        \[\,\]

    Esempio 28.  Sia

    (35)   \begin{equation*} F =\left(-\dfrac{2xz}{\left(x^2+y^2\right)^2},-\dfrac{2yz}{\left(x^2+y^2\right)^2},\dfrac{1}{x^2+y^2}+\dfrac{1}{1+z^2}\right)=\left(F_1,F_2,F_3\right). \end{equation*}

    Calcolare l’integrale di linea di seconda specie lungo l’arco di curva data dalla parametrizzazione

        \begin{equation*} \begin{cases} x=t\\ y=t^2\quad \quad \text{con}\,\, t\in \left[0,1\right]\\ z=t^3. \end{cases} \end{equation*}

        \[\,\]

    Soluzione 1.

        \[\,\]

    Sia

        \begin{equation*} r\left(t\right)=\left(t,t^2,t^3\right) \quad \text{con}\,\, t \in \left[0,1\right] \end{equation*}

    la parametrizzazione dell’arco di curva e

        \begin{equation*} r^\prime(t)=\left(1,2t,3t^2\right) \end{equation*}

    la sua derivata. \\ Restringiamo F lungo r(t)

        \begin{align*} F\left(r(t)\right)&= \left(\dfrac{-2t^4}{\left(t^2+t^4\right)^2},\dfrac{-2t^5}{\left(t^2+t^4\right)^2},\dfrac{1}{t^2+t^4}+\dfrac{1}{1+t^6}\right)=\\ &=\left(-\dfrac{2}{\left(1+t^2\right)^2},-\dfrac{2t}{\left(1+t^2\right)^2},\dfrac{1}{t^2\left(1+t^2\right)}+\dfrac{1}{1+t^6}\right) \end{align*}

    e calcoliamo il prodotto scalare tra F\left(r\left(t\right)\right) e r^\prime(t)

        \begin{align*} F\leftr\left(t\right)\right)\cdot r^\prime\left(t\right)=-\dfrac{2}{\left(1+t^2\right)^2}-\dfrac{4t^2}{\left(1+t^2\right)^2}+\dfrac{3}{\left(1+t^2\right)}+\dfrac{3t^2}{\left(1+t^6\right)} \end{align*}

    da cui

        \[\int_{\gamma}F\left(r\left(t\right)\right)\cdot &r^\prime\left(t\right)\,dt=\]

        \[&=\int_{0}^{1}\left(-\dfrac{2}{\left(1+t^2\right)^2}-\dfrac{4t^2}{\left(1+t^2\right)^2}+\dfrac{3}{\left(1+t^2\right)}+\dfrac{3t^2}{\left(1+t^6\right)}\right)\, dt\overset{*}{=}\]

        \[&\overset{*}{=}3\arctan t +\arctan t^3\bigg \vert^1_0-2\int_{0}^{1}\dfrac{1+2t^2}{1+t^2}\,dt=\]

        \[&=3\cdot \dfrac{\pi}{4}+\dfrac{\pi}{4}-2\left(\int_{0}^{1}\dfrac{2+2t^2}{\left(1+t^2\right)^2}\,dt-\int_{0}^{1}\dfrac{1}{\left(1+t^2\right)^2}\,dt \right)=\]

        \[&=\pi-2\left(2\int_{0}^{1}\dfrac{1}{1+t^2}\,dt-\int_{0}^{1}\dfrac{1}{\left(1+t^2\right)^2}\,dt\right)=\]

        \[&=\pi-4\int_{0}^{1}\dfrac{1}{1+t^2}\,dt+2\int_{0}^{1}\dfrac{1}{\left(1+t^2\right)^2}\,dt=\]

        \[&=\pi-4\arctan t \bigg \vert^1_0+2\int_{0}^{1}\dfrac{1}{\left(1+t^2\right)^2}\,dt=\]

        \[&=\pi-4\cdot \dfrac{\pi}{4}+2\int_{0}^{1}\dfrac{1}{\left(1+t^2\right)^2}\,dt=\]

        \[&=2\int_{0}^{1}\dfrac{1}{\left(1+t^2\right)^2}\,dt=I\]

    dove in * è stato usato \displaystyle\int\frac{f^\prime(t)}{1+f^2(t)}\,dt=\arctan f(t)+c. Calcoliamo I operando la sostituzione t=\tan z

        \[I&=2\int_{0}^{\frac{\pi}{4}}\dfrac{1}{\left(1+\tan^2z\right)^2}\cdot \dfrac{1}{\cos^2z}\, dz=2\int_{0}^{\frac{\pi}{4}}\cos^2 z \, dz=\]

        \[&=2\int_{0}^{\frac{\pi}{4}}\dfrac{1+\cos \left(2z\right)}{2}\, dz=z+\dfrac{\sin\left(2z\right)}{2}\bigg\vert^{\frac{\pi}{4}}_0=\dfrac{\pi}{4}+\dfrac{1}{2}.\]

    Si conclude che

        \[\,\]

        \[\boxcolorato{analisi}{\int_{\gamma}F\cdot d\bm{r}=\dfrac{1}{2}+\dfrac{\pi}{4}.}\]

        \[\,\]

    Soluzione 2.

        \[\,\]

    Il dominio di F è \mathbb{R}^3\setminus\{(0,0,z), z\in\mathbb{R}\} che non è un semplicemente connesso. Vogliamo verificare se F è irrotazionale nel suo insieme di definizione. Il rotore è dato da

        \[\nabla \times F:=\left(\dfrac{\partial F_3}{\partial y}-\dfrac{\partial F_2}{\partial z},\dfrac{\partial F_1}{\partial z}-\dfrac{\partial F_3}{\partial x},\dfrac{\partial F_2}{\partial x}-\dfrac{\partial F_1}{\partial y}\right)\]

    dove

        \[&\dfrac{\partial F_3}{\partial y}=-\dfrac{2y}{\left(x^2+y^2\right)^2},\qquad\dfrac{\partial F_2}{\partial z}=-\dfrac{2y}{\left(x^2+y^2\right)^2},\]

        \[&\dfrac{\partial F_3}{\partial x}=-\dfrac{2x}{\left(x^2+y^2\right)^2},\qquad\dfrac{\partial F_1}{\partial z}=-\dfrac{2x}{\left(x^2+y^2\right)^2},\]

        \[&\dfrac{\partial F_2}{\partial x}=-\dfrac{4xyz}{\left(x^2+y^2\right)^3},\qquad\dfrac{\partial F_1}{\partial y}=-\dfrac{4xyz}{\left(x^2+y^2\right)^3}.\]

    Pertanto possiamo concludere che

        \begin{equation*} \nabla \times F=\left(0,0,0\right) \end{equation*}

    e quindi F è irrotazionale, ma il suo dominio non è un insieme semplicemente connesso per cui non possiamo affermare a priori se esso sia conservativo o meno nel suo dominio. Ora osserviamo che la curva si trova in una zona del dominio che è un insieme semplicemente connesso quindi F è localmente conservativo. Calcoliamo un potenziale U

    (36)   \begin{equation*} \begin{cases} \dfrac{\partial U}{\partial x}=-\dfrac{2xz}{\left(x^2+y^2\right)^2}\\\\ \dfrac{\partial U}{\partial y}=-\dfrac{2yz}{\left(x^2+y^2\right)^2}\\\\ \dfrac{\partial U}{\partial z}=\dfrac{1}{\left(x^2+y^2\right)}+\dfrac{1}{1+z^2}. \end{cases} \end{equation*}

    Dalla terza equazione ricaviamo

        \begin{equation*} U(x,y,z)=\dfrac{z}{x^2+y^2}+\arctan z+c_1(x,y) \end{equation*}

    e sostituendo U(x,y,z) nella seconda equazione di ?? abbiamo

        \begin{equation*} -\dfrac{2yz}{\left(x^2+y^2\right)^2}+\dfrac{\partial c_1(x,y)}{\partial y}=-\dfrac{2yz}{\left(x^2+y^2\right)^2} \quad \Leftrightarrow \quad c_1(x,y)=c_2(x) \end{equation*}

    da cui

        \begin{equation*} U(x,y,z)=\dfrac{z}{x^2+y^2}+\arctan(z)+c_2(x). \end{equation*}

    Sostituiamo U(x,y,x) nella prima equazione di ??

        \begin{equation*} -\dfrac{2xz}{\left(x^2+y^2\right)^2}+c_2^\prime(x)=-\dfrac{2xz}{\left(x^2+y^2\right)^2}\quad \Leftrightarrow \quad c_2(x)=costante \end{equation*}

    ottenendo il potenziale

        \begin{equation*} U(x,y,z)=\dfrac{z}{x^2+y^2}+\arctan z +costante. \end{equation*}

    Calcoliamo ?? sfruttando il potenziale: osserviamo che non essendo definito in (0,0,0) il potenziale, sarà necessario calcolare il suo valore in tale punto restrigendolo lungo la curva r(t) e passando ad un limite t\to0^+

        \begin{align*} \int_{\gamma}F\cdot d\bm{r}&=U(1,1,1)-\lim_{t \rightarrow 0^+}U(r(t))=\\ &=\dfrac{1}{2}+\dfrac{\pi}{4}-\lim_{t \rightarrow 0^+}\left(\dfrac{t^3}{t^2+t^4}+\arctan t^3\right)=\dfrac{1}{2}+\dfrac{\pi}{4}. \end{align*}

    Dunque si conclude nuovamente che

        \[\,\]

        \[\int_{\gamma}F\cdot d\bm{r}=\dfrac{1}{2}+\dfrac{\pi}{4}.\]

        \[\,\]

    Esempio 30.  Determinare l’area della regione finita del piano xy delimitata dalla curva chiusa di equazioni parametriche:

    (37)   \begin{equation*} \begin{cases} x=t\cos t\\ \hspace{3.4cm} \text{con}\,\, t \in \left[0,\dfrac{\pi }{2}\right]\\ y=\left(\pi - 2t \right)\sin t \end{cases}. \end{equation*}

        \[\,\]

    Soluzione.

        \[\,\]

    L’area richiesta può essere espressa come segue

    (38)   \begin{equation*} A =\iint_D 1 \, dx\,dy \end{equation*}

    dove A è l’area della regione finita del piano xy delimitata dalla curva chiusa che abbiamo chiamato D. Applichiamo il teorema di Gauss-Green

        \[\iint_{D}\left( \dfrac{\partial f_2}{\partial x}(x,y)-\dfrac{\partial f_1}{\partial y}(x,y)\right) \,dxdy=\int_{+ \partial D}\left(f_2(x,y)dy+f_1(x,y)dx\right)\]

    Facciamo notare che la parametrizzazione scelte per la curva porta a percorrerla in senso orario: pertanto dovremo aggiungere un segno meno nell’ultimo integrale. Se nella formula di Gauss Green scegliamo

        \[f_1(x,y)=0 \qquad \mbox{e} \qquad f_2(x,y)=x\]

    allora

    (39)   \begin{equation*} \iint_D1\,dx\,dy=\iint_{ D}\left( \dfrac{\partial f_2}{\partial x}(x,y)-\dfrac{\partial f_1}{\partial y}(x,y)\right) \,dxdy=- \int_{+\partial D}x\,dy \end{equation*}

    ed applicando 37 a ?? otteniamo

        \[\int_{+\partial D}x\,dy =\int_{0}^{\frac{\pi}{2}}\left(t\cos t\cdot \dfrac{dy}{dt}\right)\,dt\]

    dove

        \[\dfrac{dy}{dt}=-2\sin t + (\pi-2t)\cos t\]

    e pertanto

        \[\int_{+\partial D}x\,dy & =\int_{0}^{\frac{\pi}{2}}t\cos t \left(-2\sin t + (\pi-2t)\cos t \right) \, dt=\]

        \[& = \underbrace{\int_{0}^{\frac{\pi}{2}}-t\sin(2t)\,dt}_{I_1}+\underbrace{\int_{0}^{\frac{\pi}{2}}\left(\pi t-2t^2 \right)\cos^2t\,dt}_{I_2}.\]

    Calcoliamo separatamente I_1 e I_2: per il primo si ha

        \[&I_1=\dfrac{\cos(2t)}{2}t\bigg \vert^\frac{\pi}{2}_0-\int_{0}^{\frac{\pi}{2}}\dfrac{\cos(2t)}{2}\,dt=-\dfrac{\pi}{4}-\dfrac{\sin(2t)}{4}\bigg \vert^\frac{\pi}{2}_0=-\dfrac{\pi}{4},\]

    mentre per il secondo

        \[I_2&=\int_{0}^{\frac{\pi}{2}}\left(\pi t-2t^2 \right)\left( \dfrac{1+\cos(2t)}{2}\right) \,dt=\left( \dfrac{t}{2}+\dfrac{\sin(2t)}{4}\right) \left( \pi t-2t^2\right) \bigg \vert^\frac{\pi}{2}_0\]

        \[&-\int_{0}^{\frac{\pi}{2 }}\left(\dfrac{t}{2}+\dfrac{\sin(2t)}{4}\right)\left(\pi-4t \right)\,dt=\]

        \[&=\dfrac{\pi}{4}\left(\dfrac{\pi^2}{2}-2\cdot \dfrac{\pi^2}{4} \right)-\left(\dfrac{t^2}{4}-\dfrac{\cos(2t)}{8} \right)\left(\pi-4t \right)\bigg \vert^{\frac{\pi}{2}}_0\]

        \[&+\int_{0}^{\frac{\pi}{2}}\left(\dfrac{t^2}{4}-\dfrac{\cos(2t)}{8} \right)(-4)\,dt=\]

        \[&=-\left(\dfrac{\pi^2}{16}+\dfrac{1}{8}\right) \left( \pi -2\pi \right)-\dfrac{1}{8}\pi -4 \left(\dfrac{t^3}{12}-\dfrac{\sin(2t)}{16} \right) \bigg \vert^\frac{\pi}{2}_0=\]

        \[&=\pi\left(\dfrac{\pi^2}{16}+\dfrac{1}{8}\right) -\dfrac{1}{8}\pi -4\left(\dfrac{\pi^3}{12\cdot 8} \right)=\dfrac{\pi^3}{48}\]

    da cui ?? diventa

        \[\int_{+\partial D}x\,dy =\int_{0}^{\frac{\pi}{2}}\left(t\cos t\cdot \dfrac{dy}{dt}\right)\,dt= I_1 + I_2 = \dfrac{\pi^3}{48}-\dfrac{\pi}{4}.\]

    Concludiamo che l’area della regione cercata è

        \[\,\]

        \[\boxcolorato{analisi}{A=-\left(\dfrac{\pi^3}{48}-\dfrac{\pi}{4}\right) .}\]

        \[\,\]

        \[\,\]


    1. Si utilizza la seguente relazione: se

          \[f:A \subset \mathbb{R} \rightarrow \mathbb{R},\,\]

      è una funzione derivabile nel suo dominio, e detta

          \[y=\ln \left \vert f(x) \right \vert\]

      la sua derivata risulta

          \[y^\prime=\sigma(f(x))\dfrac{f^\prime(x)}{f(x)},\]

      dove \sigma indica la funzione segno. Applichiamo la relazione ad U(x,y), notando che su \gamma, U>0.

  •     \[\,\]

        \[\,\]

    Appendici

        \[\,\]

        \[\,\]

    Leggi...

        \[\,\]

    Un esempio difficile

        \[\,\]

    Proponiamo di seguito un esempio con un grado di difficoltà superiore rispetto agli esempi trattati nel resto del manuale.

        \[\,\]

    Esercizio.  Sia data la seguente forma differenziale

        \[\omega&=\]

        \[&=\dfrac{-2\left(x^2y+4y^3+4x^3+xy^2\right)}{\left(4x^2+y^2\right)\left(x^2+4y^2\right)}\,dx+\dfrac{2\left(x^3+4xy^2-16x^2y-4y^3\right)}{\left(4x^2+y^2\right)\left(x^2+4y^2\right)}\,dy=\]

        \[& = F_1 \; dx + F_2 \; dy\]

    Calcolare

  • \int_{\gamma_1}\omega dove il sostegno di \gamma_1 è il segmento che congiunge i punti (0,1) e (0,2);
  • \int_{\gamma_2}\omega dove il sostegno di \gamma_2 è un quadrato di vertici (-1,-1),(1,-1),(1,1) e (-1,1).
  •     \[\,\]

    Soluzione.

        \[\,\]

    Un’analisi delle funzioni presenti suggerisce di riscriverle scomponendole in frazioni più semplici: infatti si ha per la prima

        \[F_1&=-\dfrac{2x\left(4x^2+y^2\right)}{\left(4x^2+y^2\right)\left(x^2+4y^2\right)}-\dfrac{2y\left(x^2+4y^2\right)}{\left(4x^2+y^2\right)\left(x^2+4y^2\right)}=\]

        \[&=-\dfrac{2x}{x^2+4y^2}-\dfrac{2y}{\left(4x^2+y^2\right)}\]

    mentre per la seconda

        \[F_2&=\dfrac{2x\left(x^2+4y^2\right)}{\left(4x^2+y^2\right)\left(x^2+4y^2\right)}-\dfrac{8y\left(4x^2+y^2\right)}{\left(4x^2+y^2\right)\left(x^2+4y^2\right)}=\]

        \[&=\dfrac{2x}{4x^2+y^2}-\dfrac{8y}{x^2+4y^2}.\]

    Calcoliamo le derivate miste

        \[\dfrac{\partial F_2}{\partial x}&=\dfrac{2}{4x^2+y^2}-\dfrac{2x(8x)}{(4x^2+y^2)^2}+\dfrac{2x(8y)}{(x^2+4y^2)^2}\]

        \[&=\dfrac{-8x^2+2y^2}{(4x^2+y^2)^2}+\dfrac{16xy}{(x^2+4y^2)^2}\]

    e

        \[\dfrac{\partial F_1}{\partial y}&=-\dfrac{2}{4x^2+y^2}+\dfrac{2y(2y)}{(4x^2+y^2)^2}+\dfrac{2x(8y)}{(x^2+4y^2)^2}\]

        \[&=\dfrac{-8x^2+2y^2}{(4x^2+y^2)^2}+\dfrac{16xy}{(x^2+4y^2)^2}\]

    pertanto

        \begin{equation*} \dfrac{\partial F_2}{\partial x}=\dfrac{\partial F_1}{\partial y} \end{equation*}

    quindi \omega è chiusa. Il dominio di \omega è \mathbb{R}^2\setminus \{(0,0)\} che non è un insieme semplicemente connesso pertanto non possiamo applicare il teorema di Poincaré, ma il sostegno della curva \gamma_1 si trova in una zona del dominio che è un insieme semplicemente connesso, pertanto per il medesimo teorema \omega è localmente esatta. Calcoliamo un potenziale U che deve soddisfare le seguenti condizioni

    (40)   \begin{equation*} \begin{cases} F_1=\dfrac{\partial U}{\partial x}\\\\ F_2=\dfrac{\partial U}{\partial y} \end{cases} \end{equation*}

    Dalla prima equazione di 40 abbiamo

        \begin{align*} U(x,y)&=\int \left(-\dfrac{2x}{x^2+4y^2}-\dfrac{2y}{\left(4x^2+y^2\right)}\right)\,dx=\\ &=-\int\dfrac{2x}{x^2+4y^2}\,dx-\int \dfrac{2y}{\left(4x^2+y^2\right)}\,dx=\\ &=-\ln\left(x^2+4y^2\right)-\arctan\left(\dfrac{2x}{y}\right)+c(y). \end{align*}

    Sostituiamo ora U nella seconda equazione di 40

        \begin{align*} \dfrac{2x}{4x^2+y^2}-\dfrac{8y}{x^2+4y^2}=\dfrac{\partial U}{\partial y}&=-\dfrac{8y}{x^2+4y^2}+\dfrac{\frac{1}{2x}}{\left(\frac{y}{2x}\right)^2+1}+\dfrac{dc}{dy}\left(y\right)=\\ &=-\dfrac{8y}{x^2+4y^2}+\dfrac{2x}{4x^2+y^2}+\dfrac{dc}{dy}\left(y\right) \end{align*}

    da cui

        \begin{equation*} c(y)=\text{costante} \end{equation*}

    pertanto

        \begin{equation*} U(x,y)=-\ln\left(x^2+4y^2\right)-\arctan\left(\dfrac{y}{2x}\right)+\text{costante}. \end{equation*}

    Si conclude che

        \[\,\]

        \[\boxcolorato{analisi}{\int_{\gamma_1}\omega=U(0,2)-U(0,1)=-\ln 16+\ln 4=-\ln 4. }\]

        \[\,\]

    Rappresentiamo il sostegno di \gamma_2 percorso in senso antiorario e il polo (0,0) nell’origine

        \[\,\]

        \[\,\]

        \[\,\]

    Rendered by QuickLaTeX.com

        \[\,\]

        \[\,\]

        \[\,\]

    Abbiamo dunque

        \begin{align*} \omega=\\ &=\underbrace{-\dfrac{2x}{x^2+4y^2}\,dx-\dfrac{8y}{x^2+4y^2}\,dy}_{\omega_1}+\underbrace{-\dfrac{2y}{4x^2+y^2}\,dx+\dfrac{2x}{4x^2+y^2}\,dy}_{\omega_2}. \end{align*}

    dove \omega_1 e \omega_2 sono chiuse (la verifica è lasciata al lettore – è una conseguenza dell’esattezza precedente). Calcoliamo \int_{\gamma_2}\omega_1 applicando il Teorema di Gauss Green. Poniamo per comodità \omega_1=Pdx+Qdy. Consideriamo il seguente percorso chiuso

        \[\,\]

        \[\,\]

        \[\,\]

    Rendered by QuickLaTeX.com

        \[\,\]

        \[\,\]

        \[\,\]

    dove il sostegno di \gamma_3 è \{(x,y)\in\mathbb{R}^2:\,x^2+4y^2=1\}. Poiché il dominio D racchiuso tra il quadrato e l’ellisse non contiene il polo della forma \omega_1 applicando Gauss-Green abbiamo che

        \begin{equation*} \int_{\gamma_2}\omega_1-\int_{\gamma_3}\omega_1=\iint_D \left(\dfrac{\partial P}{\partial y}-\dfrac{\partial Q}{\partial x}\right)\,dxdy=0 \end{equation*}

    e pertanto

        \begin{equation*} \int_{\gamma_2}\omega_1=\int_{\gamma_3}\omega_1. \end{equation*}

    Parametrizziamo il sostegno di \gamma_3

        \begin{equation*} \gamma_3(\theta)=\left(\cos\theta,\dfrac{1}{2}\sin \theta\right)\quad \text{con}\,\, \theta \in [0,2\pi] \end{equation*}

    da cui

        \begin{align*} \int_{\gamma_2}\omega_1&=\int_{\gamma_3}\omega_1=\\ &=\int_{0}^{2\pi}\left(-2\cos\theta \left(-\sin \theta\right)-8\left(\dfrac{1}{2}\sin\theta\right)\left(\dfrac{1}{2}\cos\theta\right)\right)\,d \theta=\\ &=\int_{0}^{2\pi}-4\cos\theta \sin \theta \,d \theta=\dfrac{\cos^2\theta}{2}\bigg\vert^{2\pi}_0=0. \end{align*}

    Calcoliamo ora \int_{\gamma_2}\omega_2 applicando nuovamente il Teorema di Gauss Green. Consideriamo ora il seguente percorso chiuso

        \[\,\]

        \[\,\]

        \[\,\]

    Rendered by QuickLaTeX.com

        \[\,\]

        \[\,\]

        \[\,\]

    dove \gamma_4 ha come sostegno \{(x,y)\in \mathbb{R}^2:\,4x^2+y^2=1\} e per il Teorema di Gauss Green, ragionando come il caso precedente, abbiamo

        \begin{equation*} \int_{\gamma_2}\omega_2 = \int_{\gamma_4}\omega_2. \end{equation*}

    Parametrizziamo il sostegno di \gamma_4

        \begin{equation*} \gamma_4\left(\theta\right)=\left(\dfrac{1}{2}\cos\theta , \sin \theta \right)\quad \text{con}\,\, \theta \in [0,2\pi] \end{equation*}

    da cui

        \begin{align*} \int_{\gamma_2}\omega_2 &= \int_{\gamma_4}\omega_2=\\ &=\int_{0}^{2\pi}\left(-2\sin \theta \left(-\dfrac{1}{2}\sin \theta \right)+2\left(\dfrac{1}{2}\cos\theta\right)\cos \theta \right)\,d \theta = \\ &=\int_{0}^{2\pi}\left(\cos^2 \theta +\sin^2 \theta \right)\,d \theta =2\pi. \end{align*}

        \[\,\]

    boxcolrato

        \[\int_{\gamma_2}\omega=2\pi.\]

        \[\,\]

        \[\,\]

    Forme differenziali e Funzioni speciali

        \[\,\]

        \[\,\]

    In questa sezione proponiamo un esercizio che lega le forme differenziali alle funzioni speciali.

        \[\,\]

    Esempio 31.  Calcolare il seguente integrale definito:

        \[\alpha\int_{0}^{\frac{\pi}{2}} (\sin x)^\beta (\cos x)^{\gamma} \, dx \quad \mbox{con}\quad \alpha \, \mbox{,} \, \beta \, \mbox{,} \, \gamma \in \mathbb{R}\]

    dove \alpha e \beta sono due numeri reali non nulli tali che valga la seguente relazione:

        \[\alpha (1- \cos x )-2x^\beta=o(x^2) \quad \mbox{per} \,\,x\rightarrow 0\]

    \gamma è il valore del seguente integrale doppio :

        \[\gamma=\dfrac{10}{8(\sqrt{3}\pi-3)}\iint_\mathcal{D} \left((x+3)(\sqrt{3}+y) \right)dx \,dy\]

    essendo \mathcal{D} è l’intersezione dei tre cerchi centrati in (1,0),(-1,0) e (0,\sqrt{3}) e ciascuno di raggio 2.

        \[\,\]

    Soluzione.

        \[\,\]

    Determiniamo \alpha e \beta applicando la definizione di “o piccolo”

        \[\begin{aligned} &\lim_{x \rightarrow 0}\dfrac{\alpha\left(1-\cos x \right)-2x^\beta}{x^2}=\\ &=\lim_{x \rightarrow 0}\dfrac{\frac{1}{2}\alpha x^2-\frac{\alpha x^4}{24}-2x^\beta +o \left(x^{min(4,\beta)} \right)}{x^2}=0 \Leftrightarrow\\\\ &\Leftrightarrow \begin{cases} \dfrac{\alpha}{2}=2\\\\ \beta=2\\ \end{cases} \quad \Leftrightarrow \quad \begin{cases} \alpha=4\\ \beta=2 \end{cases} \end{aligned}\]

    Concludiamo che i valori cercati di \alpha e \beta sono:

        \[\alpha=4,\,\beta=2.\]

    Ora determiniamo il valore di \gamma .

        \[\]

    Le equazioni delle circonferenze sono:

        \[&\gamma_1:(x-1)^2+y^2=4\]

        \[&\gamma_2:(x+1)^2+y^2=4\]

        \[&\gamma_3:x^2+(y-\sqrt{3})^2=4\]

    e rappresentiamo la regione \mathcal{D} nella seguente figura:

        \[\,\]

        \[\,\]

        \[\,\]

    Rendered by QuickLaTeX.com

        \[\,\]

        \[\,\]

        \[\,\]

    Chiamiamo

        \[f(x,y)=(x+3)(\sqrt{3}+y)=\sqrt{3}x+xy+3\sqrt{3}+3y\]

    e posto

        \[g(x,y)=\sqrt{3}x+xy \quad e \quad n(x,y)=3\sqrt{3}+3y\]

    riscriviamo l’integrale doppio come segue

        \[\iint_\mathcal{D} f(x,y) dx \,dy= \iint_\mathcal{D} g(x,y)dx \,dy+\iint_\mathcal{D} n(x,y) dx \,dy.\]

    Osserviamo che g(-x,y)=-g(x,y) quindi g è dispari rispetto alla variabile x. Inoltre \mathcal{D} è simmetrico rispetto all’asse y, dunque

        \[\iint_\mathcal{D} g(x,y)dx \,dy=0\]

    Non ci resta che calcolare \iint_\mathcal{D} n (x,y) \, dx \,dy. Applichiamo il teorema di Gauss-Green

        \[\iint_\mathcal{D} \left(3\sqrt{3}+3y \right)dx\,dy= \int_{+\partial \mathcal{D}}\left( 3\sqrt{3}+3y\right)x\,dy=\int_{+\partial \mathcal{D}}\omega\]

    Parametrizziamo la frontiera di \mathcal{D}

        \[\begin{aligned} &\gamma_1(\theta)=\left(-1+2\cos \theta,2 \sin \theta \right) \quad \text{con}\,\,\theta \in \left[0,\dfrac{\pi}{3}\right]\\ &\gamma_2(\theta)=\left(1+2\cos \theta,2 \sin \theta\right)\quad \text{con}\,\,\in \left[\dfrac{2\pi}{3},\pi\right]\\ &\gamma_3(\theta)=(2\cos \theta,\sqrt3+2\sin \theta)\quad \text{con}\,\,\in \left[\dfrac{4\pi}{3},\dfrac{5\pi}{3}\right]. \end{aligned}\]

    Quindi

    (41)   \begin{equation*} \int_{+\partial D}\omega=\int_{\gamma_1}\omega+\int_{\gamma_2}\omega+\int_{\gamma_3}\omega\\ \end{equation*}

    Calcoliamo i singoli termini della 41

        \[\begin{aligned} \int_{\gamma_1}\omega&=\int_{0}^{\frac{\pi}{3}}\left((3\sqrt{3}+3(2\sin \theta))(-1+2\cos \theta)(2\cos \theta) \right)\,d\theta=\\ & = \int_{0}^{\frac{\pi}{3}} \left(-6\sqrt{3}\cos \theta +12\sqrt{3}\cos^2\theta -12 \sin \theta \cos \theta + 24 \sin \theta \cos^2 \theta \right)\,d\theta=\\ & = -6\sqrt{3} \sin \theta \bigg\vert_{0}^{\frac{\pi}{3}}+6\sqrt{3} \left(\theta +\sin \theta \cos \theta \right) \bigg\vert_{0}^{\frac{\pi}{3}} +3 \cos (2\theta)\bigg\vert_{0}^{\frac{\pi}{3}} - 8 \cos^3 \theta \bigg\vert_{0}^{\frac{\pi}{3}}=\\ & = -9+6\sqrt{3}\left(\dfrac{\pi}{3} + \dfrac{\sqrt{3}}{4}\right) +3 \left(-\dfrac{1}{2}-1\right)- 8 \left(\dfrac{\sqrt{1}}{8}-1\right)=\\ & = 2\pi\sqrt{3} -2. \end{aligned}\]

        \[\begin{aligned} \int_{\gamma_2}\omega&=\int_{\frac{2}{3}\pi}^{\pi}\left((3\sqrt{3}+3(2\sin \theta))(1+2\cos \theta)(2\cos \theta) \right)\,d\theta=\\ & = \int_{\frac{2}{3}\pi}^{\pi} \left(6\sqrt{3}\cos \theta +12\sqrt{3}\cos^2\theta +12 \sin \theta \cos \theta + 24 \sin \theta \cos^2 \theta \right)\,d\theta=\\ & = 6\sqrt{3} \sin \theta \bigg\vert_{\frac{2}{3}\pi}^{\pi}+6\sqrt{3} \left(\theta+\sin\theta \cos \theta\right) \bigg\vert_{\frac{2}{3}\pi}^{\pi} -3 \cos (2\theta)\bigg\vert_{\frac{2}{3}\pi}^{\pi} - 8 \cos^3 \theta \bigg\vert_{\frac{2}{3}\pi}^{\pi}=\\ & = 6\sqrt{3} \left(-\dfrac{\sqrt{3}}{2}\right) + 6 \sqrt{3} \left(\pi-\dfrac{2}{3}\pi + \dfrac{\sqrt{3}}{4}\right) - 3 \left(1+\dfrac{1}{2}\right)-8\left(-1+\dfrac{1}{8}\right)= \\ & = -9 + 2\sqrt{3}\pi + \dfrac{9}{2}-\dfrac{9}{2}+8-1 =\\ & = 2\sqrt{3}\pi-2.\\\\ &\\ \int_{\gamma_3}\omega&=\int_{\frac{4\pi}{3}}^{\frac{5\pi}{3}}\left((3\sqrt{3}+3(\sqrt{3}+2\sin \theta ))(2\cos \theta)^2 \right)\,d\theta=\\ & = \int_{\frac{4\pi}{3}}^{\frac{5\pi}{3}}\left(24\sqrt{3} \, \cos^2 \theta + 24 \, \sin \theta \, \cos^2 \theta \right)\,d\theta=\\ & = 12 \sqrt{3} \left(\theta + \sin \theta \cos\theta\right) \bigg\vert_{\frac{4\pi}{3}}^{\frac{5\pi}{3}} -8 \, \cos^3 \theta \bigg\vert_{\frac{4\pi}{3}}^{\frac{5\pi}{3}} = \\ & = 4\sqrt{3} \, \pi - 18 - 2 =\\ & = 4 \left(\sqrt{3} \, \pi - 5\right). \end{aligned}\]

    La 41 diventa così

        \[\int_{+\partial \mathcal{D}}\omega=2\pi\sqrt{3}-2+2\pi\sqrt{3}-2+4\left(\sqrt{3}\pi-5 \right)=8\left(\sqrt{3}\pi-3 \right)\]

    e concludiamo che

        \[\iint_\mathcal{D} f(x,y) \; dx \,dy= \iint_\mathcal{D} g(x,y) \; dx \,dy+\iint_\mathcal{D} n(x,y) \; dx \,dy=8\left(\sqrt{3}\pi-3 \right).\]

    In particolare

        \[\gamma= \dfrac{10}{8(\sqrt{3}\pi-3)}\iint_D \left((x+3)(\sqrt{3}+y) \right)dx \,dy=10.\]

    Non ci resta che calcolare

    (42)   \begin{equation*} \alpha\int_{0}^{\frac{\pi}{2}} (\sin x)^\beta (\cos x)^{\gamma}dx= 4\int_{0}^{\frac{\pi}{2}} (\sin x)^2 (\cos x)^{10}dx. \end{equation*}

    Ricordiamo che la funzione Beta di Eulero è definita come

    (43)   \begin{equation*} B\left(a,b\right)=2\int_{0}^{\frac{\pi}{2}}\left((\sin \theta)^{2a-1}(\cos \theta)^{2b-1} \right)d\theta=\dfrac{\Gamma(a)\Gamma(b)}{\Gamma(a+b)} \end{equation*}

    Confrontando 42 con 43 si può impostare il seguente sistema

        \[\displaystyle\begin{cases} 2a-1=2\\\\ 2b-1=10 \end{cases}\quad \Leftrightarrow\quad \displaystyle\begin{cases} a=\dfrac{3}{2}\\\\ b=\dfrac{11}{2} \end{cases}\]

    ottenendo11

        \[\begin{aligned} 4\int_{0}^{\frac{\pi}{2}} (\sin x)^2 (\cos x)^{10}dx & =2\dfrac{\Gamma\left(\dfrac{3}{2}\right)\Gamma\left(\dfrac{11}{2}\right)}{\Gamma(7)}=\\ &= 2 \;\dfrac{\left(\dfrac{1}{2}\right)\left(\dfrac{9}{2}\right)\left(\dfrac{7}{2}\right)\left(\dfrac{5}{2}\right)\left(\dfrac{3}{2}\right)\left(\dfrac{1}{2}\right)\left(\Gamma\left(\dfrac{1}{2}\right)\right)^2}{6!}=\\ & =\dfrac{21}{512}\left(\Gamma\left(\dfrac{1}{2}\right)\right)^2\\ \end{aligned}\]

    Ricordando che

        \[\Gamma\left(\frac{1}{2}\right)=\sqrt{\pi}\]

    abbiamo

        \[4\int_{0}^{\frac{\pi}{2}} (\sin x)^2 (\cos x)^{10}dx=\dfrac{21}{512}\left(\Gamma\left(\frac{1}{2}\right)\right)^2= \dfrac{21}{512}\pi.\]

    Si conclude che

        \[\,\]

        \[\boxcolorato{analisi}{4\int_{0}^{\frac{\pi}{2}} (\sin x)^2 (\cos x)^{10}dx= \dfrac{21}{512}\pi. }\]

        \[\,\]

        \[\,\]

    Un’applicazione alla fisica: Teorema del lavoro

        \[\,\]

        \[\,\]

    Teorema.  Dato un punto materiale di massa m soggetto a n forze, la somma del lavoro di tutte le forze agenti su di esso lungo un percorso \gamma uguaglia la variazione di energia cinetica ovvero

        \[K_f - K_0 = \sum_{k=1}^n L_k\]

    dove K_f e K_0 sono l’energia cinetica nello stato finale e nello stato iniziale, essendo K=\dfrac{1}{2}mv^2, L_k è il lavoro della forza \vec{F}_k.

        \[\,\]

    Pertanto la relazione precedente si scrive come

        \[\dfrac{1}{2}mv_f^2 - \dfrac{1}{2}mv_0^2 = L_1 + \dots + L_n\]

    dove v_f e v_0 sono la velocità finale e iniziale della particella. Nota: le forze posso essere sia di natura conservativa che non conservativa. Dimostrazione. In figura 1 è rappresentato un punto materiale, soggetto a n forze, che si muove di moto vario rispetto ad un sistema fisso Oxyz lungo un percorso \gamma

        \[\,\]

        \[\,\]

        \[\,\]

    Rendered by QuickLaTeX.com

        \[\,\]

        \[\,\]

        \[\,\]

    Dalla seconda legge della dinamica abbiamo che

    (44)   \begin{equation*} \vec{F}_1 + \vec{F}_2 + \dots + \vec{F}_n = \dfrac{d\vec{P}}{dt} \end{equation*}

    dove \vec{P} = m\vec{v} \`{e} la quantità di moto. Ipotizzando che la massa non dipenda dal tempo, la ?? diventa

        \[\vec{F}_1 + \vec{F}_2 + \dots + \vec{F}_n = m \dfrac{d\vec{v}}{dt}.\]

    Ora ricordiamo che il lavoro di una forza è definito come segue

    (45)   \begin{equation*} L = \int_{\gamma} \vec{F} \cdot d\vec{r} = \int_{t_0}^{t_f} \vec{F}(\vec{r}(t)) \cdot \vec{r}^{\,\prime}(t) \; dt \end{equation*}

    dove \gamma è il sostegno (la traiettoria percorsa da m) di estremi A e B (vedi figura 1), \vec{r}(t)=x(t) \, \hat{x}+y(t) \, \hat{y}+z(t) \, \hat{z} è la parametrizzazione di \gamma con t \in [t_0,t_f] ed infine \vec{r}^{\,\prime}(t)=\vec{v}(t) è la velocità di m. Notiamo che \vec{r}(t_0)=A e \vec{r}(t_f)=B. La ?? puo’ essere riscritta come segue

        \[\begin{aligned} & \int_{\gamma} \left(\vec{F}_1 + \dots + \vec{F_n}\right) \cdot d\vec{r} = \int_{\gamma} m \, \dfrac{d\vec{v}}{dt} \cdot d\vec{r} \,\,\,\,\Leftrightarrow\\ & \Leftrightarrow \; \underbrace{\int_{\gamma} \vec{F}_1 \cdot d\vec{r}}_{\shortstack{$\scriptstyle{ L_1 = \text{Lavoro } }$\\ $\scriptstyle{F_1 \text{ da } A \to B }$}} + \dots + \underbrace{\int_{\gamma} \vec{F}_n \cdot d\vec{r}}_{\shortstack{$\scriptstyle{L_n = \text{Lavoro } }$\\ $\scriptstyle{ F_n \text{ da } A \to B }$}} = \int_{\gamma} m \, \dfrac{d\vec{v}}{dt} \cdot d\vec{r}\,\,\,\, \Leftrightarrow\\ & \Leftrightarrow L_1 + \dots + L_n = \underbrace{\sum_{k=1}^n L_k}_{\shortstack{\scriptsize{ Somma \, di \,}\\ \scriptsize{tutti\, i \, lavori }} } = \int_{\gamma} m \, \dfrac{d\vec{v}}{dt} \cdot d\vec{r} \end{aligned}\]

    Consideriamo ora

        \[\int_{\gamma} m \, \dfrac{d\vec{v}}{dt} \cdot d\vec{r}\]

    e poichè m non dipende da d\vec{r}, allora possiamo portarla fuori dall’integrale

        \[\int_{\gamma} m \, \dfrac{d\vec{v}}{dt} \cdot d\vec{r} = m \int_{\gamma} \dfrac{d\vec{v}}{dt} \cdot d\vec{r}\]

    Da 45 abbiamo

        \[m \int_{\gamma} \dfrac{d\vec{v}}{dt} \cdot d\vec{r} = m \int_{t_0}^{t_f} \dfrac{d\vec{v}}{dt} \cdot \vec{v} \, dt\]

    Ora ricordiamo che \vec{v} \cdot \vec{v} = v^2 e derivando quest’ultima da ambo i membri

        \[\dfrac{d(\vec{v} \cdot \vec{v})}{dt} = \dfrac{d(v^2)}{dt} \Leftrightarrow \vec{v} \cdot \dfrac{d\vec{v}}{dt} + \dfrac{d\vec{v}}{dt} \cdot \vec{v}= 2 v \; \dfrac{dv}{dt}\]

    Poiché il prodotto scalare è una forma bilineare simmetrica otteniamo

        \[\vec{v} \cdot \dfrac{d\vec{v}}{dt}+ \dfrac{d\vec{v}}{dt} \cdot \vec{v} = \vec{v} \cdot \dfrac{d\vec{v}}{dt} + \vec{v} \cdot \dfrac{d\vec{v}}{dt} = 2 \vec{v} \cdot \dfrac{d\vec{v}}{dt} = 2v \; \dfrac{dv}{dt} \Leftrightarrow \vec{v} \cdot \dfrac{d\vec{v}}{dt} = v \dfrac{dv}{dt}\]

    quindi

        \[m \int_{t_0}^{t_f} \dfrac{d\vec{v}}{dt} \, \cdot \, \vec{v} \; dt = m \int_{t_0}^{t_f} v \; \dfrac{dv}{dt} \; dt = m\dfrac{v^2(t)}{2} \bigg\vert_{t_0}^{t_f} = \dfrac{m}{2} v^2(t_f)-\dfrac{m}{2} v^2(t_0)= \dfrac{m}{2} \left(v_f^2 - v_0^2\right)\]

    da cui

        \[\sum_{k=1}^n L_k = \dfrac{1}{2} \, m \, \left(v_f^2 - v_0^2\right).\]

    Formulando la relazione precedente in forma discorsiva, possiamo affermare quanto segue: la somma di tutti i lavori agenti su un punto materiale che va da un punto A a B lungo un percorso \gamma (vedi figura 1) uguaglia la variazione di energia cinetica . Osserviamo che il teorema del lavoro può essere espresso anche in funzione della quantità di moto come segue

        \[\sum_{k=1}^n L_k = \dfrac{1}{2} \, m \, \left(v_f^2 - v_0^2\right)= \dfrac{1}{2m} \left(p_f^2 - p_0^2\right).\]

        \[\,\]

    Dalle definizioni già fornite in precedenza di potenziale (non inteso in senso fisico) per un campo di forze conservativo, segue direttamente in fisica il concetto di energia potenziale associata ad una forza: se infatti la forza \vec{F} risulta conservativa e quindi è determinata da un potenziale U (nel senso della nostra definizione 2) per cui \vec{F}=\nabla{U} allora si definisce in fisica energia potenziale la quantità E_{pot}=-U.

        \[\,\]

    Lemma.  Il lavoro compiuto da una forza conservativa su un punto materiale m uguaglia la variazione di energia potenziale cambiata di segno, ovvero

        \[L_{A \to B} = -\Delta E_{pot}=E_{pot}(A)-E_{pot}(B)\]

    In particolare esso non dipende dalla traiettoria percorsa da m (vedi figura 1) ma solamente dai punti iniziale e finale .

        \[\,\]

    Dimostrazione Lemma.

        \[\,\]

    Sia \vec{r}(t) con t \in [t_0,t_f] la parametrizzazione di \gamma, la traiettoria percorsa da un punto materiale m che si muove da A a B (vedi figura 1) soggetto solo a forze conservative. Dalla 45 si ha che

        \[\begin{aligned} L=&\int_\gamma \vec{F}\cdot d\vec{r}=\int_{t_0}^{t_f}\vec{F}(\vec{r}(t)) \cdot {\vec{r}\,}^\prime(t) \,dt=\int_{t_0}^{t_f}\nabla U(\vec{r}(t)) \cdot {\vec{r}\,}^\prime(t) \,dt=\\ &=\int_{t_0}^{t_f}\dfrac{d}{dt}\left( U(\vec{r}(t))\right)\,dt= U(\vec{r}(t_f))-U(\vec{r}(t_0))=-\Delta E_{pot}\\ \end{aligned}\]

        \[\,\]

    Nel caso in cui la parametrizzazione del sostegno \gamma sia di classe C^1 a tratti si ragiona nello stesso modo in ogni tratto in cui è regolare e si sommano i vari contributi per ottenere il risultato.

        \[\,\]

    Conservazione dell’energia meccanica

        \[\,\]

    Nell’ipotesi che su tale punto agiscano solo forze di natura conservativa, dal teorema dell’energia-lavoro e dal Lemma si ha che

    (46)   \begin{equation*} \begin{aligned} & K_f - K_0 =-\Delta E_{pot} =E_{pot, \,0} -E_{pot,\,f} \Leftrightarrow \\ & \Leftrightarrow \boxed{K_f + E_{pot, \, f} = K_0 + E_{pot,\,0}} \end{aligned} \end{equation*}

    Il risultato 46 rappresenta un concetto fondamentale in fisica ovvero che su un punto materiale sul quale agiscono solo forze conservative, si conserva l’energia meccanica totale E=K+E_{pot}. Questo teorema prende il nome di conservazione dell’energia meccanica.

        \[\\\\\]

    Nota Il teorema delle forze vive spesso viene chiamato anche teorema del lavoro-energia o dell’energia cinetica.

        \[\,\]

        \[\,\]


    1. Si ricordi che

          \[\Gamma(x+1)=x\Gamma(x),\,x\in\mathbb{R}\qquad \Gamma(n+1)=n!,\, n\in\mathbb{N}.\]

  •  

    Tutta la teoria di analisi matematica

    Leggi...

    1. Teoria Insiemi
    2. Il metodo della diagonale di Cantor
    3. Logica elementare
    4. Densità dei numeri razionali nei numeri reali
    5. Insiemi Numerici \left(\mathbb{N},\, \mathbb{Z},\, \mathbb{Q}\right)
    6. Il principio di induzione
    7. Gli assiomi di Peano
    8. L’insieme dei numeri reali: costruzione e applicazioni
    9. Concetti Fondamentali della Retta Reale: Sintesi Teorica
    10. Costruzioni alternative di \mathbb{R}
    11. Binomio di Newton
    12. Spazi metrici, un’introduzione
    13. Disuguaglianza di Bernoulli
    14. Disuguaglianza triangolare
    15. Teoria sulle funzioni
    16. Funzioni elementari: algebriche, esponenziali e logaritmiche
    17. Funzioni elementari: trigonometriche e iperboliche
    18. Funzioni goniometriche: la guida essenziale
    19. Teorema di Bolzano-Weierstrass per le successioni
    20. Criterio del rapporto per le successioni
    21. Definizione e proprietà del numero di Nepero
    22. Limite di una successione monotona
    23. Successioni di Cauchy
    24. Il teorema ponte
    25. Teoria sui limiti
    26. Simboli di Landau
    27. Funzioni continue – Teoria
    28. Il teorema di Weierstrass
    29. Il teorema dei valori intermedi
    30. Il teorema della permanenza del segno
    31. Il teorema di Heine-Cantor
    32. Il teorema di esistenza degli zeri
    33. Il metodo di bisezione
    34. Teorema ponte versione per le funzioni continue
    35. Discontinuità di funzioni monotone
    36. Continuità della funzione inversa
    37. Teorema delle contrazioni o Teorema di punto fisso di Banach-Caccioppoli
    38. Teoria sulle derivate
    39. Calcolo delle derivate: la guida pratica
    40. Teoria sulle funzioni convesse
    41. Il teorema di Darboux
    42. I teoremi di de l’Hôpital
    43. Teorema di Fermat
    44. Teoremi di Rolle e Lagrange
    45. Il teorema di Cauchy
    46. Espansione di Taylor: teoria, esempi e applicazioni pratiche
    47. Polinomi di Taylor nei limiti: istruzioni per l’uso
    48. Integrali definiti e indefiniti
    49. Teorema fondamentale del calcolo integrale (approfondimento)
    50. Integrali ricorsivi
    51. Formule del trapezio, rettangolo e Cavalieri-Simpson
    52. Teoria sugli integrali impropri
    53. Funzioni integrali – Teoria
    54. Introduzione ai numeri complessi – Volume 1 (per un corso di ingegneria — versione semplificata)
    55. Introduzione ai numeri complessi – Volume 1 (per un corso di matematica o fisica)
    56. Serie numeriche: la guida completa
    57. Successioni di funzioni – Teoria
    58. Teoremi sulle successioni di funzioni
      1. 58a. Criterio di Cauchy per la convergenza uniforme
      2. 58b. Limite uniforme di funzioni continue
      3. 58c. Passaggio al limite sotto il segno di integrale
      4. 58d. Limite uniforme di funzioni derivabili
      5. 58e. Piccolo teorema del Dini
      6. 58f. Procedura diagonale e teorema di Ascoli-Arzela
    59. Serie di funzioni – Teoria
    60. Serie di potenze – Teoria
    61. Serie di Fourier – Teoria e applicazioni
    62. Integrali multipli — Parte 1 (teoria)
    63. Integrali multipli — Parte 2 (teoria e esercizi misti)
    64. Regola della Catena — Teoria ed esempi.
    65. Jacobiano associato al cambiamento di coordinate sferiche
    66. Guida ai Massimi e Minimi: Tecniche e Teoria nelle Funzioni Multivariabili
    67. Operatore di Laplace o Laplaciano
    68. Teoria equazioni differenziali
    69. Equazione di Eulero
    70. Teoria ed esercizi sulla funzione Gamma di Eulero
    71. Teoria ed esercizi sulla funzione Beta
    72. Approfondimento numeri complessi
    73. Diverse formulazioni dell’assioma di completezza
    74. Numeri di Delannoy centrali
    75. Esercizi avanzati analisi

    Tutte le cartelle di Analisi Matematica

    Leggi...

    1. Prerequisiti di Analisi
      1. Ripasso algebra biennio liceo
      2. Ripasso geometria analitica
      3. Ripasso goniometria e trigonometria
      4. Errori tipici da evitare
      5. Insiemi numerici N,Z,Q,R
      6. Funzioni elementari
      7. Logica elementare
      8. Insiemi
    2. Successioni
      1. Teoria sulle Successioni
      2. Estremo superiore e inferiore
      3. Limiti base
      4. Forme indeterminate
      5. Limiti notevoli
      6. Esercizi misti Successioni
      7. Successioni per ricorrenza
    3. Funzioni
      1. Teoria sulle funzioni
      2. Verifica del limite in funzioni
      3. Limite base in funzioni
      4. Forme indeterminate in funzioni
      5. Limiti notevoli in funzioni
      6. Calcolo asintoti
      7. Studio di funzione senza derivate
      8. Dominio di una funzione
      9. Esercizi misti Funzioni
      10. Esercizi misti sui Limiti
    4. Funzioni continue-lipschitziane-holderiane
      1. Teoria sulle Funzioni continue-lipschitziane-holderiane
      2. Continuità delle funzioni
      3. Continuità uniforme
      4. Teorema degli zeri
      5. Esercizi sul teorema di Weierstrass senza l’uso delle derivate
    5. Calcolo differenziale
      1. Derivate
      2. Calcolo delle derivate
      3. Retta tangente nel calcolo differenziale
      4. Punti di non derivabilità nel calcolo differenziale
      5. Esercizi sul teorema di Weierstrass con l’uso delle derivate
      6. Studio di funzione completo nel calcolo differenziale
      7. Esercizi teorici nel calcolo differenziale
      8. Metodo di bisezione
      9. Metodo di Newton
    6. Teoremi del calcolo differenziale
      1. Teoria sui Teoremi del calcolo differenziale
      2. Teorema di Rolle
      3. Teorema di Lagrange
      4. Teorema di Cauchy
      5. Teorema di De L’Hôpital
    7. Calcolo integrale
      1. Integrale di Riemann
      2. Integrali immediati
      3. Integrale di funzione composta
      4. Integrali per sostituzione
      5. Integrali per parti
      6. Integrali di funzione razionale
      7. Calcolo delle aree
      8. Metodo dei rettangoli e dei trapezi
      9. Esercizi Misti Integrali Indefiniti
      10. Esercizi Misti Integrali Definiti
    8. Integrali impropri
      1. Teoria Integrali impropri
      2. Carattere di un integrale improprio
      3. Calcolo di un integrale improprio
    9. Espansione di Taylor
      1. Teoria Espansione di Taylor
      2. Limiti di funzione con Taylor
      3. Limiti di successione con Taylor
      4. Stime del resto
    10. Funzioni integrali (Approfondimento)
      1. Teoria Funzioni integrali (Approfondimento)
      2. Studio di funzione integrale
      3. Limiti con Taylor e De L’Hôpital
      4. Derivazione di integrali parametrici (Tecnica di Feynmann)
    11. Numeri Complessi
      1. Teoria Numeri complessi
      2. Espressioni con i numeri complessi
      3. Radice di un numero complesso
      4. Equazioni con i numeri complessi
      5. Disequazioni con i numeri complessi
      6. Esercizi misti Numeri complessi
    12. Serie numeriche
      1. Teoria Serie numeriche
      2. Esercizi Serie a termini positivi
      3. Esercizi Serie a termini di segno variabile
      4. Esercizi Serie geometriche e telescopiche
    13. Successioni di funzioni
      1. Teoria Successioni di funzioni
      2. Esercizi Successioni di funzioni
    14. Serie di funzioni
      1. Teoria Serie di funzioni
      2. Esercizi Serie di funzioni
    15. Serie di potenze
      1. Teoria Serie di potenze
      2. Esercizi Serie di potenze
    16. Serie di Fourier
      1. Teoria Serie di Fourier
      2. Esercizi Serie di Fourier
    17. Trasformata di Fourier
      1. Teoria Trasformata di Fourier
      2. Esercizi Trasformata di Fourier
    18. Funzioni di più variabili
      1. Teoria Funzioni di più variabili
      2. Massimi e minimi liberi e vincolati
      3. Limiti in due variabili
      4. Integrali doppi
      5. Integrali tripli
      6. Integrali di linea di prima specie
      7. Integrali di linea di seconda specie
      8. Forme differenziali e campi vettoriali
      9. Teorema di Gauss-Green
      10. Integrali di superficie
      11. Flusso di un campo vettoriale
      12. Teorema di Stokes
      13. Teorema della divergenza
      14. Campi solenoidali
      15. Teorema del Dini
    19. Equazioni differenziali lineari e non lineari
      1. Teoria equazioni differenziali lineari e non lineari
      2. Equazioni differenziali lineari e non lineari del primo ordine omogenee
    20. Equazioni differenziali lineari
      1. Del primo ordine non omogenee
      2. Di ordine superiore al primo,a coefficienti costanti,omogenee
      3. Di ordine superiore al primo,a coefficienti costanti,non omogenee
      4. Di Eulero,di Bernoulli,di Clairaut,di Lagrange e di Abel
      5. Non omogenee avente per omogenea associata un’equazione di Eulero
      6. Sistemi di EDO
    21. Equazioni differenziali non lineari
      1. A variabili separabiliO
      2. A secondo membro omogeneo
      3. Del tipo y’=y(ax+by+c)
      4. Del tipo y’=y(ax+by+c)/(a’x+b’y+c’)
      5. Equazioni differenziali esatte
      6. Mancanti delle variabili x e y
      7. Cenni sullo studio di un’assegnata equazione differenziale non lineare
      8. Di Riccati
      9. Cambi di variabile: simmetrie di Lie
    22. Analisi complessa
      1. Fondamenti
      2. Funzioni olomorfe
      3. Integrale di Cauchy e applicazioni
      4. Teorema della curva di Jordan e teorema fondamentale dell’Algebra
      5. Teorema di inversione di Lagrange
      6. Teorema dei Residui
      7. Funzioni meromorfe
      8. Prodotti infiniti e prodotti di Weierstrass
      9. Continuazione analitica e topologia
      10. Teoremi di rigidità di funzioni olomorfe
      11. Trasformata di Mellin
    23. Equazioni alle derivate parziali
      1. Equazioni del primo ordine
      2. Equazioni del secondo ordine lineari
      3. Equazioni non-lineari
      4. Sistemi di PDE
    24. Funzioni speciali
      1. Funzione Gamma di Eulero
      2. Funzioni Beta,Digamma,Trigamma
      3. Integrali ellittici
      4. Funzioni di Bessel
      5. Funzione zeta di Riemann e funzioni L di Dirichlet
      6. Funzione polilogaritmo
      7. Funzioni ipergeometriche
    25. Analisi funzionale
      1. Misura e integrale di Lebesgue
      2. Spazi Lp,teoremi di completezza e compattezza
      3. Spazi di Hilbert,serie e trasformata di Fourier
      4. Teoria e pratica dei polinomi ortogonali
      5. Spazi di Sobolev
    26. Complementi
      1. Curiosità e approfondimenti
      2. Compiti di analisi
      3. Esercizi avanzati analisi
    27. Funzioni Convesse

    Tutti gli esercizi di geometria

    In questa sezione vengono raccolti molti altri esercizi che coprono tutti gli argomenti di geometria proposti all’interno del sito con lo scopo di offrire al lettore la possibilità di approfondire e rinforzare le proprie competenze inerenti a tali argomenti.

    Strutture algebriche.




    Risorse didattiche aggiuntive per approfondire la matematica

    Leggi...

    • Math Stack Exchange – Parte della rete Stack Exchange, questo sito è un forum di domande e risposte specificamente dedicato alla matematica. È una delle piattaforme più popolari per discutere e risolvere problemi matematici di vario livello, dall’elementare all’avanzato.
    • Art of Problem Solving (AoPS) – Questo sito è molto noto tra gli studenti di matematica di livello avanzato e i partecipanti a competizioni matematiche. Offre forum, corsi online, e risorse educative su una vasta gamma di argomenti.
    • MathOverflow – Questo sito è destinato a matematici professionisti e ricercatori. È una piattaforma per domande di ricerca avanzata in matematica. È strettamente legato a Math Stack Exchange ma è orientato a un pubblico con una formazione più avanzata.
    • PlanetMath – Una comunità collaborativa di matematici che crea e cura articoli enciclopedici e altre risorse di matematica. È simile a Wikipedia, ma focalizzata esclusivamente sulla matematica.
    • Wolfram MathWorld – Una delle risorse online più complete per la matematica. Contiene migliaia di articoli su argomenti di matematica, creati e curati da esperti. Sebbene non sia un forum, è una risorsa eccellente per la teoria matematica.
    • The Math Forum – Un sito storico che offre un’ampia gamma di risorse, inclusi forum di discussione, articoli e risorse educative. Sebbene alcune parti del sito siano state integrate con altri servizi, come NCTM, rimane una risorsa preziosa per la comunità educativa.
    • Stack Overflow (sezione matematica) – Sebbene Stack Overflow sia principalmente noto per la programmazione, ci sono anche discussioni rilevanti di matematica applicata, specialmente nel contesto della scienza dei dati, statistica, e algoritmi.
    • Reddit (r/Math) – Un subreddit popolare dove si possono trovare discussioni su una vasta gamma di argomenti matematici. È meno formale rispetto ai siti di domande e risposte come Math Stack Exchange, ma ha una comunità attiva e molte discussioni interessanti.
    • Brilliant.org – Offre corsi interattivi e problemi di matematica e scienza. È particolarmente utile per chi vuole allenare le proprie capacità di problem solving in matematica.
    • Khan Academy – Una risorsa educativa globale con lezioni video, esercizi interattivi e articoli su una vasta gamma di argomenti di matematica, dalla scuola elementare all’università.